Misunderstandings about determinism

April 27, 2015 • 11:15 am

I’m giving a talk on free will at the Imagine No Religion 5 meetings in Vancouver in June (the event appears to have sold out already). As you might imagine, it’s a complicated task. But one of the things I want to cover are the misunderstandings people often have about physical determinism, which include these:

Determinism leads to fatalism.

Determinism leads you to act badly, including cheating and criminality.

Determinism means that nobody should be punished for anything, as they had no choice in what they did.

Determinism means that we can predict in practice everything that will happen. (Determinism doesn’t mean that we humans always have all the information we need to predict things, and of course some things are fundamentally unpredictable if they’re affected by quantum phenomena.)

(By the way, if you’ve encountered other misconceptions, please note them in the comments.)

But by far the most common misconception I’ve encountered is this one:

Determinism means that it’s useless to try to influence the behavior of others (or to change anything else), as those results are predetermined.

Frankly, I’m surprised at this one, as it’s easily dispelled.  My common rejoinder is this: if you meet a friendly dog, and then begin kicking it every time you see it, the dog will soon begin to avoid you. You’ve influenced that dog’s behavior. And so it is with people and institutions.  Determinism does not mean “unalterable”; it means that what happens obeys the laws of physics.

Now it is also true that whether or not I kick the dog is itself determined, and that behavior is the ineluctable result of my genetic and environmental history, which themselves are determined. It’s truly an infinite regress. But none of this shows that the impact of environments on us—how people treat us, talk to us, and other things we experience—cannot alter our behavior. For those influences feed into our wet onboard computer—our brain—and affect its output. What move one makes in a chess match, for instance, will affect the response of a computer that you might be playing against. With computers, as with us, if the input differs, so will the output. And, since our brains have evolved by natural selection, if you alter an input, the output will (like the skittish dog) likely be adaptive.

Yet I constantly receive emails and comments on this site from people who feel that they’ve produced a Professor Ceiling Cat “gotcha” moment by informing me that I am somehow tacitly endorsing free will by either implying that behaviors can be changed or by even trying to change them (or persuading others to do likewise). Often the emails or comments are rude or verging on it. Such is the nature of many people who see discourse as an intellectual game to be won.

But those “corrections” are misguided. Certainly my own efforts to, say, change the behavior of creationists or onlookers by mocking or correcting them is an ineluctable result of my genes and upbringing. But that doesn’t mean that I’m being hypocritical or inconsistent in what I say. I have no free choice about what I do, but what I do can still affect others. (Of course, all that, save some quantum indeterminacy, is programmed into the Universe!)

Here are some examples of people purporting to point out my mistakes on this issue:

First, an email by someone telling me that my efforts to change Paul Nelson’s mind were inconsistent with my views on free will.  (Actually I wasn’t trying to influence Nelson, which is futile: I was directing my comments towards readers and those on the fence, trying to educate them):

Dear Dr. Coyne:

It was determined by the laws of physics that Paul Nelson would hold this position and make this video.
He couldn’t do other.

Sincerely,
Daniel [last name redacted]

*******

Here’s another email I got yesterday from a friendly reader who wanted clarification about the same post, clarification that I provided:
Jerry,
“God, I despise Liars for Jesus. They should be mocked, reviled and refuted. Be my guest in the comments.”
We both know that Paul Nelson has no choice in what he does, free will is nonexistent.  The above sentence clearly implies a moral judgment regarding Paul.  How can you reconcile what you said with what you know to be true regarding libertarian free will (or any other kind for that matter)?

*******

Here’s a comment from reader David Ellinghaus on my post “Dzhokhar Tsarnaev and the death penalty“:

I am firmly opposed to the death penalty myself. However, this is a truly mad argument. The idea that nobody can be held morally accountable for anything that they do is a dangerous one. Arguing that people do not have choices – that their actions are predetermined is a slippery slope. According to this line of thought, people cannot and should not be expected to make changes or attempt to make changes in their behaviour. Furthermore, is there any point in trying to improve society when things are simply fated to happen? Just an insane argument with very insidious connotations.

Secondly, in reference to a particular point you made in this article, how can punishment be a deterrent if people would have carried out their actions anyway? It is logically fallacious to suggest that imprisonment would deter somebody whose actions are already predetermined.

I trust that the average reader here, even if she’s a compatibilist, can see the fallacy of this email. No, I don’t believe in “moral accountability,” which to me implies that one can choose freely. (Indeed, the survey of Sarkissian et al. shows that many people connect a belief in determinism with an absence of moral responsibility.) But I do believe in responsibility or accountability, by which I mean that a person who did a good or bad act is the entity who performed that act, and should be treated accordingly, especially for punishment. Punishment should, I believe, be exacted for social good, and can involve rehabilitation, deterrence, and keeping people who are dangerous out of society to prevent further damage. Those are all compatible with determinism, and need involve no “moral” opprobrium. But if you want to call society’s code of conduct “rules of morality,” I don’t have a strong objection.

As for there being “no point in trying to improve society when things are simply fated to happen,” we already know the fallacy of that. To say that it’s an “insane argument” or “logically fallacious” is simply rudeness piled on top of misunderstanding.

*******

Finally, reader Explorer commented in part on my post “More nonsense at NPR about God

. . . . I find the need to both argue that there is no free will, while criticizing people as if they have it, to be a hilarious invocation of only embracing an argument when it suits one’s needs. It’s like watching creationists who embrace any bit of science which might support them, while being willing to ignore it when it disproves their views.

In this case, as well as other numerous topics where there are odious behaviors which I do believe are due to the actor’s choices (the orthodox Jews not wanting to sit next to women, for example), I don’t understand how the criticisms of the the choices fit in with pointing out there was no real choice involved.

Personally, I believe that theoretically perfect prediction of a choice is not the same as a choice not being made. It’s just funny to constantly read someone who does argue that choice doesn’t really exist while simultaneously criticizing those choices.

This again is a rude and arrogant comment, one that went too far over the line of civility (thus removing “Explorer” from further discourse). It’s also mistaken. One can indeed affect people’s behavior by criticizing it, as I’ve mentioned a gazillion times above—even if those bad behaviors were themselves determined.  It’s just like reprogramming a computer so it gives a different output when you feed it the same input.

__________

Added note: Had Explorer written the same comment in a civil manner, making the identical points (saying something like “I don’t understand how you comport determinism with the idea that you can change people’s behavior. I don’t think those can be reconciled”, then he/she would not have been removed from the commentariat. With the increase in readership here, I see an increasing number of people who come over and just begin whaling away, not paying attention to Da Roolz. And I can’t urge people to “read the Roolz” every day. Some people get second chances, or are rebuked privately, but I expect discourse on this site to remain civil, with readers being polite to the host and to other readers. I may not always catch transgressions but, like Maru, I do my best.

277 thoughts on “Misunderstandings about determinism

  1. Hmm. I think I smell a third book coming from Jerry. It will (my prediction) be an elaboration on, or an accompaniment to, Sam Harris’s book on free will. I’ll place my pre-order now for delivery in 2018.

  2. Closely related to your last one: determinism does not mean we must abandon agency-based language, metaphors, and descriptions. Not only might that be impossible (maybe its deterministically certain that we will keep talking that way), but they are very useful shorthand and probably reasonably accurate given the minimal knowledge about internal human brain states we have. Thus even if we recognize that somebody deterministically committed a crime, it is still useful and productive to consider whether they intended to commit a crime, whether it was a crime of passion or not, etc. Yes, “intent” and “crime of passion” freight in a lot of agency, but to say the same thing without agency-based language would take a lot more work for very little decision-making benefit.

      1. I do not consider myself a compatibilist. There is (IMO) a significant difference in saying “talking as if people had agency is linguistically useful” and saying “people have agency in a way consistent with physics.”

        A flagella is a rotor. Using that sort of language does not mean I’m claiming creationism is compatible with science or empirical observation. It means ‘rotor’ is sometimes a useful description of a biological structure. I can use that language and still think creationism is wrongity wrong wrong. Don’t you agree? So I can discuss whether someone ‘meant’ to do a crime and still say that compatibilism is wrongity wrong wrong.

        1. But what property or phenomenon is the “convenient only” language of “intent” meant to describe?

          Acknowledging that there is some property of conscious matter that can be conveniently described as “intent” (and that other types of matter don’t display this property) is a big part of the compatibilist project.

          1. But what property or phenomenon is the “convenient only” language of “intent” meant to describe?

            A fully deterministic and inevitable pattern of brain activity consisting of prior planning and prior action related to successfully performing the criminal action. How is this inevitable pattern compatible with any notion of free will? What’s free about it?

          2. How is this inevitable pattern compatible with any notion of free will? What’s free about it?

            At the risk of rehashing oodles of previous threads, look up the term “free” in a dictionary. I’d bet that none of the meanings that you find means “ignores the laws of physics”. There are lots of sorts of freedom in a deterministic world.

            By the way, I’d be interested in what you think is “wrongity wrong” about compatibilism.

          3. “Degrees of freedom” is a perfectly well-defined concept in physics, and complex systems typically have more degrees of freedom than simple ones. This is precisely what arch-compatibilist Dennett means when he says that “freedom evolves”. Our brains our the most complex systems on the planet, hence the most free, in a well-defined, non-spooky sense.

          4. These are the dimensions that can change when an outside force is applied. This has nothing to do with choices.

          5. Your “degrees of freedom” is what is typically meant by the common phrase, “all else being equal.”

            But, in reality, all else is not equal — which is why Jerry and I and others of like mind reject the notion that all else is actually equal.

            “All else being equal” is a great tool, a wonderful map. But the fatal mistrake of compatibilists lies in thinking that, because the map doesn’t have any restrictions on it, reality must not, either.

            We not only know that the map is not complete, we know that its fundamental nature — that you can lay it flat and fold it up and put it in your pocket — is radically and incompatibly different from what it maps.

            When a compatibilist manages to take the entire Earth, fold it up, and slip it into her pocket, I’ll agree that we really do have something resembling “free will.” But until then?

            b&

          6. Your “degrees of freedom” is what is typically meant by the common phrase, “all else being equal.”

            Well no, it isn’t. If one says that a diatomic gas molecule has more “degrees of freedom” than a monatomic molecule, and thus that its specific heat is higher, that is nothing to do with “all else being equal”, whatever you mean by that phrase, which I’m not at all sure.

          7. In addition to what Coel and Gregory write about how the term “free” can apply to physical, deterministic systems, I’d argue that saying

            “a fully deterministic and inevitable pattern of brain activity consisting of prior planning and prior action”

            is where our colloquial notion of “intent” originates demonstrates compatibilist thinking. Brains give rise to intent. Other forms of matter do not. A big part of the compatibilist project is to say “this is an important distinction”. It seems to me a big part of the incompatibilist project is to say it isn’t. But I think pointing out that all matter ultimately reduces to mindless subatomic interactions is a truism of limited utility. I’m reminded of Pinker explaining how meaningless it would be to try to talk about WWII in terms of particle physics.

      2. Yes. If you think certain conscious entities can have “intent”, or if you think there’s a distinction worth noting between the actions of an individual who, not realizing the gun is loaded, accidentally shoots someone and an individual who plans to murder someone, then shoots him or her, you’re espousing some compatibilist ideas.

        1. Yes, there’s a distinction, and no that is not espousing compatibilism. In both cases the result was determined, in the murderer he was conditioned to do the crime, which was inevitable, and so where is the “freedom” in any of this.

          Now you’re not only redefining free will with a compatibilist view, but you’re trying to tell ME that I’m a compatibilist in a world of purely determined actions? Sorry, that won’t fly. I could tell you that since you see both actions as determined, you are an incompatibilist.

          1. Well, I’m glad that a distinction is being recognised. But from the point of view of the universe as a whole, or from the point of view of physics, it is surely not an important one, if from those points of view, it exists at all. But why do most people feel that the distinction is important and points to a genuine difference, and why are the consequences of this distinction regarded as important? And what really is gained by insisting that both actions are in the end determined (as no doubt they are)? What does it add to our understanding of these things?

          2. The distinction is important socially because it’s pointless to prosecute an earthquake or a crazy person for the damage they cause. There is some utility in prosecuting morally competent agents, because their behavior is susceptible to correction.

            People feel that the distinction is important because biological and cultural evolution have trained them to feel that way, precisely because of its social utility.

          3. Thank you, Gregory, but I did know that it’s pointless to prosecute earthquakes or crazy people (though just the other week, as I recall, a seriously brain-damaged man, was executed for a killing, probably in Texas)! And I am glad to see that you provide a REASON for the distinction despite italicising ‘feel’, as though feeling somehow trumped any rationale. Either the distinction is important or it is not; and it is not important from the point of view of the universe, but it is important in the perspective of human life, where it makes a difference.

          4. Sorry, but I’m not sure what you’re getting at here. The universe is nonsentient and doesn’t have a point of view. Importance is a human construct, and makes sense only in a human context.

            You asked why people feel the distinction is important. I took the question at face value and tried to answer, both in terms of why it is important, and why people feel it’s important (since that’s how you phrased it). If you didn’t intend it as a serious question and weren’t actually looking for an answer, accept my apologies and we’ll let it drop.

          5. Sorry if things were not clear. What I was objecting to was the way important distinctions such as that between the point of viewlessness of the universe and the meaningful context of human life seem to be collapsed in the the kind of determinism espoused by some.The questions were largely rhetorical, except for the last, since describing human life in terms of physical determinism, assuming it can be done, does not seem conducive to understanding very much about it.

    1. eric,

      I believe your post does not adequately capture the problems facing the incompatibilist.

      If an incompatibilist wants to influence another person’s behavior via making an argument it should be coherent. (Reasoned Arguments presume rational, consistency-demanding parties on both sides).

      So say the incompatibilist wants to make an argument to influence me to change my course of action; from my current vengeance-born conception of blame I use to support the death penalty,** to a more compassionate incarceration approach based on removing blame/moral responsibility.

      In order to make sense, both of those courses of actions have to be presented as OPTIONS. That I’m currently selecting one course of action, where an ALTERNATE course of action is possible. In other words, it entails I have a “choice” in a very real sense of: “I could do otherwise.”

      If not, if the incompatibilist is recommending I do otherwise, while refusing to acknowledge I could do otherwise, it’s like recommending an impossible option, which is incoherent.

      So the options left to the incompatibilist are either:

      1. Acknowledge a concept of “choice” in which it is true to that we DO have a choice, true that I COULD do otherwise (which also entails I could HAVE done otherwise). And hence his prescription will be cogent and have truth value.

      Or alternately:

      2. Refuse any such robustly true notion of true “choice” and “real ability to do otherwise.” And then say essentially: I may be using the typical language of optinos, choice-assumption and prescription in my argument, but I don’t really MEAN those terms as presented. They are just place-holders for what I really mean if I am to explain myself further.

      Option 2 is pretty much what you are getting at in the above, I believe. The question the incompatibilist has to answer, if he goes with 2 is this:

      Ok, drop the illusory/convenient/place-holder speech. And make a cogent case for what I ought to do using whatever truthful language you see fit instead.

      No incompatibilist seems to be up to doing this, that I have encountered anyway.

      The reason is, I believe, that it’s pretty much impossible. Because our general conception of options/choices/prescriptions take the form they do because it is a necessary form for describing truths about the world.

      (Nor have I seen a coherent incompatibilist version of option 1…that doesn’t essentially cede the compatibilist rational.)

      Vaal

      **(Just using an example: I don’t support the death penalty)

      (Also, as for option 1, I have many times here defended a conception of “choice/could do otherwise” that has truth value given determinism, and which is used in daily life).

      1. Acknowledge a concept of “choice” in which it is true to that we DO have a choice, true that I COULD do otherwise (which also entails I could HAVE done otherwise).

        Again, only in the construction of, “all else being equal.” Which it never actually is.

        That’s the entire sticking point in this discussion.

        Compatibilists (etc.) are pointing to the map, noting that it’s not only flat but can be folded neatly and put in your pocket, and therefore concluding that the Earth itself is not only flat but foldable.

        Incompatibilists are trying to tell the compatibilists that, no, the Earth is neither flat nor foldable, but the map is still quite useful nonetheless so long as you don’t extrapolate beyond its intended function.

        b&

        1. Ben,

          Sorry, I just don’t understand what you are saying. I don’t think I (or anyone else) gets the relevance of this “all else being equal” phrase you keep using.

          But more to the point, I think it’s telling you aren’t answering the challenge either 😉

          1. To repeat an example I used earlier today, “All else being equal, I’d prefer an apple to an orange. But having seen a worm just crawl out of that apple, I’ll have the orange instead.”

            Your “choices” only make sense in that they ignore all other influences. But keep peeling away the layers, and it’s “other influences” all the way ‘down, such that all else is never actually equal — and it’s the “all else” that’s actually what determines what you do.

            Even your preferences for apples over oranges. You think that, too, isn’t as much a result of an unbreakable causal chain as the worm’s appearance in the apple being presented to you?

            It’s very helpful to make certain abstractions in your analysis of things. By stating a preference for apples, you’re generally saying that your average experience for apples has been more pleasing than your average experience for oranges — or, perhaps even more nuanced, your average experience for apples in situations similar to the one you’re in right now has been preferable. But that’s just an abstract heuristic. In this particular instance here, it’s not just a choice between an apple and an orange; it’s a choice between a wormy apple and an orange — which is hardly any choice at all, no?

            Ultimately, all your choices are a matter of “There really wasn’t any other way I could have chosen at the time, what with the selection I had to pick from and the way I was thinking at the time.”

            Alter the circumstances, the “all else,” and your choice is correspondingly altered. So if the variable is the circumstances, not you…what sense does it make to say that you, not the variables, are in control?

            b&

          2. Hi again Ben,

            From what I can tell, your apple/orange example is just a re-statement of the idea the specific state of affairs/causes that exist at any particular time, we could not have done otherwise.

            But that’s just Determinism 101, conceded long ago at the outset between incompatibilists and compatibilists, so it doesn’t move the conversation any further.

            The question is, given we live in that context, we still deliberate about courses of action, selecting between options, and when making the case to one another about which courses of action to take, we appeal to options, choices, ability to do otherwise etc.

            Now, if you say that Determinism 101 necessarily renders such concepts entirely illusory, then nothing we actually say in such context makes sense. You have pulled the rug out from under not just any prescriptive case you want to make to others; you’ve pulled it out of the very forms of inference science has used, from which you are getting specific evidence for determinism in the first place.

            But if there is truth in saying “you should choose X over Y” or “you have the option of choosing X over Y” then you have to make sense of it in the deterministic context. Spell it out. I’ve done this many times from the compatibilist viewpoint.

            So let’s take your apple/orange example.
            Let’s say there’s an apple and an orange on the counter that someone has presented as my “options” for a snack. I tell you I am going to choose the apple. But you know there is a worm in it.

            How will you make the case that I ought to INSTEAD take the action of choosing the orange?

            What case can you make to me that does not rely explicitly or implicitly on the assumption I “could do otherwise” in order to make sense? How will I get from whatever facts you lay out to “therefore I OUGHT to choose the orange instead?”

            Quick…I’m reaching for the apple 😉

            Vaal

          3. The question is, given we live in that context, we still deliberate about courses of action, selecting between options, and when making the case to one another about which courses of action to take, we appeal to options, choices, ability to do otherwise etc.

            Yes. We do.

            And each and every facet of those deliberations is similarly subjected to, to continue the theme, all else not being equal. Whether or not you consider a particular factor depends on whether or not you’ve ever even encountered it as a possibility, if you remember the fact that you encountered it, if you’re properly weighting its significance…and that level, in turn depends a great deal on the physiology and the chemistry and the physics and so on.

            All you’re doing with your “given we live in that context” is drawing a line in the “all else,” below which you’re assuming that all the “all else” really is equal.

            Again, that’s an incredibly useful analytical tool, as your continuation of the analogy demonstrates. Does wonders for helping us figure out which is what.

            But to claim that it means that there really is some sort of choice, “free will,” or what-not based on the results of the analysis of where you happened to draw the line?

            You’re right back to claiming that the Earth really is flat, after all, because, now that you’ve ironed out all the wrinkles you can’t see the not-flatness any more. Misses the point entirely.

            The Earth is not flat nor foldable, even if it’s often useful to pretend that it is when you want to make your way across town to the drugstore and all you have to guide you is the map in your pocket.

            We don’t actually make choices, even if it’s often useful to pretend that we do when confronted with an apple and an orange to eat.

            b&

          4. Ben,

            Again, you are speaking in generalities, avoiding the challenge I offered you.

            How would you REASON with me, making the case that I ought to eat the orange instead of the apple with the worm in it?

            You know there’s a worm in the apple, that I wouldn’t like to eat an apple with a worm in it.

            So tell me what you’d say to me, that makes the rational case I ought to eat the orange instead. It would be a piece of cake if you simply resort to everyday reasoning to do this; e.g. “Vaal, I see you’ve expressed the intent to eat the apple. However, I spotted a worm in that apple earlier. Whereas there is no worm in the orange. If you don’t want to eat the fruit with a worm in it, you should eat the orange instead of the apple.”

            Simple. Reasonable. Useful. True.
            And it “works” both in logical coherence, and in terms of persuasive power, on the assumption that I “could choose otherwise” than I am intending at the moment.

            Note that the logic works in both directions – toward the past as well as the future. If I end up choosing the apple and spit it out in disgust, I could afterward be told “Given you were informed of the worm in the apple and didn’t want a wormy fruit, you SHOULD have chosen the orange instead.”

            This is the everyday logic we use to “learn from past mistakes” and it works on the same concept as prescriptions for future action.

            Now, again, the challenge to you is to make a persuasive case for me to change my course of action from choosing to eat the wormy apple, to eating the fresh orange.

            How exactly do you do this? Please write what you would say to me, if it isn’t the type of everyday “choice-assuming” language
            that would normally get the job done.

          5. Vaal, just as I’m going to grab a map when I want to navigate somewhere and the fact that its geometry bears not the slightest semblance to the Earth’s actual geometry be damned…in everyday conversation, yes, of course, I’m going to use the imprecise but incredibly useful “all else being equal” models of our inescapably dualistic language.

            But this isn’t a causal everyday conversation. This is a setting where we’re debating the geometry of the Earth, and of course the Earth isn’t flat and foldable like the map.

            So why on Earth would you insist that it really is? What could it possibly gain you to note how useful your map is and how you just couldn’t do without it as part of your defense of your proposition that the Earth really is flat?

            All I’m doing is saying, “You do realize, don’t you, that this map isn’t nothing like the Earth, right?”

            What you’re doing is saying, “But don’t you see how we just used this map to find our way to the café? That just proves that the map is the same shape as the Earth — flat and foldable! Why, you admitted as much when you shoved the map in your pocket!”

            b&

          6. Ben,

            There’s a reason you aren’t taking up the challenge. It will immediately show you have no recourse for a logical argument, without the explicit or implicit assumption that alternative actions are possible.

            As to your map example, you’ve got the answer right in your hands. Why is a map “useful” at all? It is only useful insofar as it conveys *some form of truth* about the terrain you want to navigate. If it didn’t you wouldn’t use the map in the first place.

            A map may not *be* the terrain, but if it accurately tells you that the distance from your current location to where you want to go is one mile, then it is telling you something TRUE about the real world, about the real terrain. Right? It’s not an ‘illusory’ truth. It’s not a “useful fiction” it is a TRUTH delivered via the conceptual method of understanding real terrain: true information conveyed in the form of maps. You can acknowledge both that the map isn’t the terrain, while also understanding the map conveys TRUTHS about the terrain.

            Now, what if someone insisted that maps don’t “really” convey truths about the terrain? Why? Because the only “real truth” is the terrain itself. Any other form of talking about the terrain is false, illusory. Well, that would be an utterly impractical notion. It would make a complete hash of the logic of creating maps in the first place. If we couldn’t convey truths via maps, the whole project makes no sense.

            But not only would it make maps illogical; the person making this claim would be left trying to explain how it is we have used maps to RELIABLY, SUCCESSFULLY NAVIGATE REAL TERRAIN in the first place. How in the world could that be happening, if maps themselves were not conveying “truth?”

            Wouldn’t make sense.

            And this is where we are with the language of “choice” “options” “alternatives” “could do otherwise.” We use these concepts all the time, like the maps, to understand the world, successfully navigate it, to make logical arguments to other people as to which options to choose, which we use to improve our ability to navigate the options in the world. If this conceptual framework were simply “illusory” and not itself conveying truth, like the map example, how do you explain the fact we use these concepts all the time to think about our actions, to persuade others to change courses of action, and that this conceptual scheme WORKS to navigate the world?
            How do you explain that you can not present any ALTERNATIVE to this conceptual scheme?

            So in contrast to your characterization, the compatibilist is not saying the map (our concepts of alternative action) *is* equivalent to the terrain. Rather we are saying the map *conveys truths about the terrain; a necessary tool to navigate the terrain*

            To say “I could choose either the apple or the orange” isn’t to mistake ourselves as outside the laws of physics, but like the map it nonetheless conveys a truth about me, in this world: about my powers in general, and using “if/then” reasoning to understand what I am capable of in this world (and to therefore predict what I’m capable of doing, and decide between alternatives based on such understanding).

            Whereas you and incompatibilists such as you seem stuck insisting that the map has to be the terrain itself in order to be “true.”

            And if that *isn’t* your position…if your position is that, like ours, maps aren’t simply fictions or illusions, but a method of conveying truths about the terrain (real world)….welcome to compatibilism 😉

          7. A map may not *be* the terrain, but if it accurately tells you that the distance from your current location to where you want to go is one mile, then it is telling you something TRUE about the real world, about the real terrain. Right?

            Of course. But that’s not how you’re using your map.

            You’re measuring the distance between the points on the map with your inch ruler and claiming that the distance between the points is about six and a quarter inches.

            You see, the flaw in your use of the analogy is that

            And this is where we are with the language of “choice” “options” “alternatives” “could do otherwise.”

            can only make sense if you ignore the actual terrain. You could walk from one side of the Grand Canyon to the other in a straight line, if you just pick up the map, fold it, and step straight from the one side to the other.

            But the actual terrain isn’t foldable like that, even though your map is.

            And, in reality, you could only “do otherwise” if none of these huge array of factors actually come into the picture. “All else being equal.”

            Yes, you could pick the apple over the orange if all else is equal. If there’s no worm coming out of the apple. If I don’t tell you that the apple is laced with cyanide. If I don’t tell you that was a joke and you believe me. And so on.

            Those are all elements missing from your map!

            Fill in your map with more and more of those elements…and, eventually, it turns out that, no, there actually isn’t a path from here to there on your map, and there’s just the one-and-only decision left that you can even theoretically make — and, sure enough, that’s the decision you actually do make.

            And I don’t know why you’re continuing to insist that I’m not taking you up on your challenge. I’ve repeatedly explained that I’ll cheerfully use the common dualistic language in everyday life, just as I’ll happily reach for that map, inaccurate and fuzzy as it is. What more could you possibly want from me for your challenge?

            b&

          8. Ben I bow to your mastery of quote indentation, which I seemed “determined” not to master.
            That said…

            “You’re measuring the distance between the points on the map with your inch ruler and claiming that the distance between the points is about six and a quarter inches.”

            That is incorrect, and it should be easy to see why that’s a misunderstanding.

            If someone were interpreting a “map” as you’ve depicted it, the results would be continually, reliably inaccurate and disastrous right? I mean if someone is looking at a map that conveys the information they are a mile away from where they need to be, but they interpret it as only six and a quarter inches, their repeated failure to navigate would vindicate your characterization. But what if they were actually using the map to SUCCESSFULLY navigate to where they need to go? Then you will have to admit you’ve somehow misinterpreted how they are using the map.

            This is what you are stuck with in terms of the compatiibilist (and everyday) use of “choice, alternatives, possibilities, could do otherwise” etc. Conceptualizing the world this way allows us to SUCCESSFULLY navigate the world. That’s what you have to explain.
            Mere asides that such concepts are either somehow “useful” doesn’t explain how they could be useful (just like you have to explain HOW a map is useful, if it weren’t conveying truth).
            Nor does asides about residual “dualistic” language. Again, you have to explain HOW this can be a successful method of thinking about, choosing and interacting with the world.

            As to the specifics of what I as a compatibilist am actually “doing” an example:
            I’m deciding between two actions tomorrow, walking to work or driving. The “me” that I am conceiving of is “me” construed over time, e.g. the same “me” that two weeks ago walked to work every day, and the same “me” that chose to drive the following week.
            So long as I conceive of “me” broadly construed like this (which is what we all do), then it’s true that “I” have been capable of driving to work, and walking to work. And insofar as there is no new physical malady reducing my abilities, and no other obstructions on the way to work, I will think “I COULD drive to work IF I wanted to” and “I COULD walk to work IF I wanted to.” So far, all of those thoughts are reasonable, and only reasonable insofar as they seem to convey truths about my abilities in the world. This is just like the map that says “1 mile to your destination.” The map isn’t the destination, but it is a way of conveying TRUTHS about the world, and this is validated by your successful use of the map to get where you want.

            Similarly, the type of deliberations I describe above when choosing between actions allow me to SUCCESSFULLY navigate the world; I can routinely predict that I CAN get to work via the action of my choice, and usually the difference in time of arrival.

            Just like the situation with the map, IF this conceptual scheme is as you depicted in the map analogy SO inaccurate, so riven with illusory thinking and untruth, you have to explain how it is that it WORKS. How does conceptualizing the world in terms of alternative possibilities work not just for me, but for you, and everyone? And if you still insist this is an illusory (hence false) method of conceiving our choices, you have to come up with one that makes more sense, or you are just lost in incoherence yourself.

            If you want to say you are falling back into dualistic language (I’m not!) then you have left to explain why it would be successful in the first place, if there weren’t something true about it. (Again, I think you are mistaken that you are using dualism when adducing the idea of “choice, ” but that your own dilemma you’ve put yourself in).

            And when you say:”inaccurate and fuzzy as it is.” Then how can an “inaccurate” framework, insofar as it does not convey truth, actually be so successful if it’s not conveying something true? And by “fuzzy,’ again, what would that mean? Sort of accurate? But…how?

            I believe your response to my earlier “challenge” pretty much has proved my point anyway.

          9. I’m deciding between two actions tomorrow, walking to work or driving.

            And then you go on to consider some set of factors which would influence your decision…and explicitly ignore other factors equally influential.

            Incorporate all factors and what choice are you left with?

            Ignore selected factors and, of course, you have a choice. But it’s those factors that actually determine the decision you make, so how can you possibly ignore them?

            b&

          10. Ben,
            So, all map makers are liars, who imply that the territory is absolutely flat, and foldable to boot? That is … an amazingly uncharitable interpretation of map makers and their maps.

          11. Cartographers are well aware of the geometry of the Earth and the fact that their maps are only the crudest approximation of it, and they make no representations otherwise. They’re not lying.

            b&

          12. Ben,

            “All else equal” means that other factors beyond your opinion of apples/oranges don’t factor in.

            For example, if you were on a game show and picking the apple meant you were going to win a million dollars, then all else would not be equal.

            It’s about selecting which factors to evaluate in your desirability algorithm. A (quite intentionally) deterministic computer program is just as capable of making the “all else equal” distinction as a human.

            Also, something that bothers me a bit about how you (and Jerry) talk about free will is that it sometimes sound like you’re maintaining a sort of dualistic version of identity, where there is some “I” beyond the physical universe such that you can ask “could I have made a different choice if the physical universe was unchanged?”.

            I’d be interested in your thoughts.

          13. “All else equal” means that other factors beyond your opinion of apples/oranges don’t factor in.

            Yes! Exactly!

            For example, if you were on a game show and picking the apple meant you were going to win a million dollars, then all else would not be equal.

            Keep going with that.

            What if you weren’t going to win $1,000,000, but $999,999? How about $999,998? How about $1? How about no money on the line, but the apple’s got a worm coming out of it and the orange looks perfect? What if circumstances are such that no other factors apply?

            Even in that last case, what did you do to ensure that no other factors applied?

            Also, something that bothers me a bit about how you (and Jerry) talk about free will is that it sometimes sound like you’re maintaining a sort of dualistic version of identity, where there is some “I” beyond the physical universe such that you can ask “could I have made a different choice if the physical universe was unchanged?”.

            I blame the inherently and inescapably dualistic nature of our language. See? I can’t even reply to your question without using such language!

            We are meat computers with recursive models of reality that include ourselves in the model. We are autonomous Turing-equivalent decision-making computational machines with a great deal of flexibility in terms of hardware and some damned sophisticated software.

            But, when it comes right down to it…just like a Rube Goldberg contraption, no matter how convoluted the pathway between input and output, all we ever do is process inputs into outputs.

            b&

      2. Your comments have been among those that have changed my mind (from IC to C) on this topic.

        But I’m not sure this is a solid line of argumentation.

        The incompatibilist would probably reply that s/he is not really presenting a target of persuasion with a choice, rather simply placing an obstacle in the path of the target’s trajectory, which obstacle the incompatibilist hopes will alter that trajectory.

        1. musical beef,

          I think you are responding to my post?
          If so:

          See my further comments to Ben.

          The incompatibilist doesn’t *really* want to say something as empty as “I’m just putting an obstacle in your way.” That may as well be an admission that all he has is noise-making, not reasons. A chimpanzee could “put something in my way too.”

          But all day long incompatibilists like everyone else want to present reasons to do things as a form of persuasion. They want to think they have good arguments. If it is just “noise that MIGHT influence someone” then that is just another way of saying “I can’t give you any actual argument/reasons to change your ways” and thus there’s no reason to listen to the incompatibilist at all. It’s entirely self-defeating, and it’s not a viable position to hold, nor does anyone actually hold such a position.

          So this “my input might affect your behavior” thing is just a red-herring. The question is whether the incompatibilist is left with being able to give “reasons to do one thing vs another thing” – which is obviously something we do all day long, and which we REQUIRE to even get along in the world. So the arguments have to be coherent, not contradictory, to be of any persuasive use. Incompatibilists don’t want to allow that their arguments are on no better ground than, say, a young earth creationist. If they did, they’ve lost the conversation.

          So the move you suggest just isn’t viable at all.

          If they are in one moment going to disavow a robust concept of alternate possibilities and “could do otherwise” and in the next make prescriptions for taking one action over another, they have to show how they can make sense of this. If they are speaking in contradictions, they aren’t going to persuade any rational person, defeating their own purpose. Which is why I’ve given the challenge I have.

          1. The incompatibilist doesn’t *really* want to say something as empty as “I’m just putting an obstacle in your way.”

            I very much doubt that’s what Jerry is doing, and I’m certain it’s not what I’m doing.

            But all day long incompatibilists like everyone else want to present reasons to do things as a form of persuasion.

            Yes. Of course.

            If you wish to think of it in computational terms, we are attempting to provide different inputs that will get incorporated into other people’s weighting factors for their internal algorithmic processing.

            And, of course, it is our own current state of our own computational systems that directs these actions.

            It’s very, very convenient to describe this process, as you have done, in terms of persuasion and choice and deciding amongst options.

            Yet, nevertheless, this map is not the territory.

            The actual territory is a gigantic Rube Goldberg contraption in which this bangs into that and knocks this other thing loose, and we are but small parts of the contraption.

            Until you truly admit that all you do is no more than the elaborate functioning of a Rube Goldberg contraption, you have not truly embraced the important bits we’ve learned about how the Universe actually functions.

            And it’s not just us!

            A droplet of water isn’t really a droplet of water, either; it’s a collection of water molecules with such-and-such an average kinetic energy that causes the electromagnetic attraction of the polar molecules to “clump” in a liquid manner…and even those molecules are just atoms, which are just configurations of quarks and electrons and gluons, and so on.

            It’s most inconvenient in most contexts to describe that drop of water at the level of subatomic particles…but any “higher-level” description that fails to acknowledge or otherwise diminishes that “lower-level” reality is, frankly, one that must be instantly discarded.

            Yes, of course. Mention it and move on — but, for the sake of all that’s unholy, don’t describe your “higher-level” abstraction as the one that’s really real or that’s really governing what’s going on or the only one that matters or what-not. It may well be the only one you care about in a certain context, but so what?

            You still can’t lay the actual Earth flat on the table, fold it up, and slip it into your pocket, no matter how much you love doing so with your map.

            b&

          2. I don’t disagree with much of this.

            I’d only say that it seems to me that incompatibilists are the ones who want to ignore certain levels of explanation, namely, the higher ones. Compatibilists agree that mindless determinism is what’s going on at a foundational level. It’s incompatibilists who call the higher levels illusory, while compatibilists say they are emergent, but real.

          3. Yes, it’s very odd. It’s as if the incompatibilist is looking at a piece of cherry pie, and saying “because in reality cherry pies are only made up of atomic particles, cherry pies don’t really exist.”

            You don’t make cherry pies go away, become fiction, simply by describing them at the atomic level.

            Incompatibilists, like duallists, seem to think freedom has to be “freedom all the way down to the bottom” in order for freedom to be “real,” like insisting cherry pies have to be “cherry pies all the way down to the bottom – cherry pies have to be made of ever smaller cherry pies – in order for cherry pies to be considered ‘real.'”

          4. It’s not so much that the higher levels are illusory, but that the compatibilists are claiming emergent properties that don’t actually emerge.

            Namely, “freedom.”

            Emergence doesn’t work like that. An emergent property is one that doesn’t even apply at all at lower levels. There’s no sense in which an electron is “wet,” nor “dry,” nor any other variation on that theme…but electrons bound to hydrogen and oxygen atoms in water molecules are part of a system called “water” in which the property of wetness emerges.

            But, for the compatibilist…there’s never any point at which anything resembling freedom emerges — for it simply can’t, as the lower levels are themselves fixed and not-free. It’d be like claiming that, yes, a glass of water is wet, but “dryness” can somehow emerge in an entire lake of wet water.

            b&

          5. Well, I think it would depend on how you define “freedom”. It’s simply an observable, empirical fact that humans have more potential to get up and do more things than a boulder does. This is an accepted conception of “freedom” (not a redefinition) and it obviously has emerged. I would agree that a contracausal, “out of nowhere” conception of “freedom” could not emerge.

          6. But do computers have freedom? They, too, have a much greater degree of motion than the sand from which they’re forged, yet they’re the archetypal example of something that lacks freedom.

            b&

          7. It seems to me it’s not out of the realm of possibility. And I’m not at all sure that computers are an accepted archetype of something that lacks freedom, unless you only mean the magic, contracausal kind of freedom. I’ve seen other compatibilist commenters grant that a sufficiently complex computer would have the kind of freedom compatibilists are talking about, which is a greater potential for a greater range of possible actions. That dies t bother me. I’m pretty sure I’ve seen you refer to our brains as meat computers.

          8. Ben wrote: “But do computers have freedom? They, too, have a much greater degree of motion than the sand from which they’re forged, yet they’re the archetypal example of something that lacks freedom.”

            I’d say it’s the opposite: computers often represent the archetypal example for exploring when increasing computational complexity produces entities that reach the status of “free/autonomous/self-directed/moral.”

            I’m sure you are aware the sci-fi literature on this is pretty vast, not to mention all the movies exploring these themes (e.g. A.I., BladeRunner, Her, and tons of others).

            As compatibilists like Dennett would put it, freedom “evolves” and thus comes in degrees.
            For essentially the same reasons the capacity to suffer, or the concept of self-consciousness/intelligence might come in degrees, with increasing neurological complexity.

            Just like identifying if a creatures is conscious may be difficult, or deciding when the minute changes from one species to another over time may require a certain amount of arbitrary categorization, none of this implies the phenomenon doesn’t exist, or can’t come in degrees.

          9. Well, perhaps Ben could explain in simple what is ‘really real’ since he seems to be the only person who understands what it is to be ‘really real’.

  3. I’m always suspicious of any comment that refers to someone’s argument as ‘hilarious’. I don’t know why, but somehow use of this adjective is invariably linked to an illogical case.

    The free will discussion is one I continue to struggle with, but am now starting to get to grips with. We actually had a BBC religious programme very recently devoted to the subject, but it was dominated by religious folks who didn’t grasp the arguments. It’s important to listen to the details on this subject before getting embroiled in argument.

    1. I agree with what you say about calling someone’s argument “hilarious”. In my experience, it inevitably precedes patronizing insult as well as (as you say) failure to grasp the argument.

    2. I watched the programme – The Big Questions right? It’s an often excellent programme, but a mainstream discussion programme that’s required to allocate guest slots to all the main religions and their vacuous votaries was never likely to get to the meat of what is a deeply subtle subject.

      It’s difficult enough just clearing away the misunderstandings people have when you tell someone you don’t believe in free will without having to deal with three or four entirely distinct interpretations of what free will actually means in the first place. Religious people have their own idea, compatibilists another, enemies of neo-Darwinism and the strawman of genetic determinism have another, even social theorists have their own. All of them were on show in the episode in question and nothing really got dealt with as a result.

      I think most people think objections to free-will come from areas like sociology and genetics – the idea that a pauper doesn’t have ‘as much’ free will as a prince, and that we are constrained by our genomes. Likewise they believe that defeating objections to the idea of free-will simply involve defeating the idea of pure genetic determinism, or dealing with social inequality, neither of which pose a problem in principle. I don’t think many people are aware of the far thornier problems that the laws of physics pose.

  4. A good post, with which I, as a compatibilist, agree.

    [Any chance of a companion “misunderstandings about compatibilism” post? :-)]

    But if you want to call society’s code of conduct “rules of morality,” I don’t have a strong objection.

    I indeed do want to! The OED agrees: “moral”: “Concerned with or derived from the code of behaviour that is considered right or acceptable in a particular society”.

    I don’t think we should hand over the word to the dualists and theists, since it works fine exactly as you have just explained.

    I also think that explaining that determinism does not mean the end of morality would help gain acceptance of determinism.

    1. My problem with “morality” is that it’s deeply connected with the notion that someone could freely have chosen to be “moral” versu “immoral”. That, at least, is the conclusion of the Sarkissian et al. study of people in four nations. I’d have to redefine what morality means for me, and then I’d be like a compatibilist!

      1. Should we consider thinking of morality as a culturally contextual set of heuristics, the expression of which, is influenced by the limits of an individual’s particular neuroanatomy, neurophysiology, and prior memory?

          1. It comes up as .nz in New Zealand. It’s not the only site to do that. I remember another one doing the same that I knew wasn’t in New Zealand – I think it was Rosa Rubicondior, but not sure.

      2. The Sarkissian et al. study is a little more complicated.

        When they asked: “If the universe is totally deterministic, are people fully morally responsible for their acts?”, their subjects tended to answer “no”.

        But when they asked about a specific case of wrong doing, e.g.: “suppose in a totally deterministic universe, where all Jack’s actions are determined, Jack robs a bank and kills a policeman, is Jack morally responsible for this?” they tend to reply “yes”.

        Which suggests that people’s thoughts on this are not a fully coherent package.

        I’d suggest that people’s day-to-day moral judgements are pretty pragmatic. So of course Jack is morally responsible, that’s how society works, and he should be put in jail.

        But if you ask them an abstract question, they then refer to their superficial reasoning about this — which is at odds with how they actually think.

        This suggests that any link between morality and libertarian FW is actually pretty weak and superficial. Asking a concrete question about a specific case is all it takes to break it.

    2. Any chance of a companion “misunderstandings about compatibilism” post?

      Perhaps a guest post by Dennett.

      1. And many we should. Like, “free will.”

        It is, after all, a perfect self-contained oxymoron. A will that is free is unmoored and directionless and without meaning. Freedom constrained by the will is no freedom at all.

        Let the religious have their married bachelors. We don’t need to burden ourselves with their own incoherence.

        b&

        1. Hey Ben,
          I messed up in that other thread, which is now closed. I wrote “X is the only possible outcome, for everyone” whereas what I meant was “Paul-does-X is the only possible (possible for anyone) outcome”. So what I actually wrote down, attributed a view to you that you obviously don’t hold. Mea culpa.

          However, now that I’ve corrected my previous horrible formulation, does it correctly represent your view?

          Anyway my view is that, given determinism, Paul-does-X is the only possible outcome for everyone but Paul. Quinn-does-Y is the only possible outcome for everyone but Quinn. Etc. But for Paul, Paul-does-X and Paul-does-Y may both be possible; and similarly for Quinn. Possibility (in a sense relevant to freedom of action) should be thought of, first and foremost, as a relation between a person and a situation.

          1. However, now that I’ve corrected my previous horrible formulation, does it correctly represent your view?

            No.

            Your presentation presumes a Platonic sort of essence to a person.

            You might prefer chocolate to vanilla. You might go with your friend who prefers vanilla over chocolate to an ice cream parlor. Keeping things simple for the moment in what, in reality, would be a much more dynamic environment, neither of you will have any choice of which ice cream to get: your nature is such that you prefer chocolate and so that’s what you get; your friend is similarly compelled to order the vanilla.

            You discover the chocolate to be decidedly underwhelming, but your friend reports that the vanilla is excellent. You each have a taste of the other’s and come to the same conclusions.

            The next night you go to the same parlor. You now have no choice but to order the vanilla, as does your friend.

            The first night, had you known that the chocolate would be so inferior to the vanilla, you would have had no choice but to order the chocolate…but you didn’t know that, so you couldn’t possibly have made that choice.

            In each and every instances, the circumstances are such that there is but one single choice for you to make, and that’s the “choice” that you actually do make.

            You can now propose all sorts of other hypothetical alterations to the situation which will result in a different choice. But, in each of those hypotheticals, the circumstances have changed and so the one-and-only choice has changed to match the circumstances.

            If I am to grant that this constitutes, “free will,” then I must also grant that a pocket calculator similarly has, “free will.” Its display can show any and all numbers with up to as many digits as the display can hold. If I press, “1 + 1,” it will show, “2”; if I press, “2 + 2,” it will show, “4.” It can freely chose to show any and every possible number, and I can demonstrate that freedom by actually causing it to show any and every possible number.

            In reality, of course, any such “choice” is a mere illusion, born of the same place as your own “choices.”

            b&

          2. Ben,
            Thanks. Let me try again. Does the following sentence represent your view:
            “Paul-does-X-at-time-t is the only possible (possible-for-anyone) outcome.”
            Where X in your example story would be ordering the chocolate ice cream, and t is that first night.

            I’m not sure what a Platonic sort of essence is, so I don’t know if I’ve implied them. My guess is that because I messed up again(!) by leaving off the at-time-t, I came off as saying that each person does one characteristic thing, all the time, as if some particular behavior were the Platonic essence for each person. I didn’t mean to do that.

            I won’t try to compare my view to yours and argue for mine, until I’ve gotten the basics of yours characterized.

          3. “Paul-does-X-at-time-t is the only possible (possible-for-anyone) outcome.” Where X in your example story would be ordering the chocolate ice cream, and t is that first night.

            Still no. That same night, somebody else orders vanilla, while you, standing right next to that person, order chocolate.

            Every situation is a function. Given a particular individual, that individual’s history and knowledge, that individual’s observations, and all the rest, there is a single possible outcome. Change the individual, the individual’s history, knowledge, observations, or anything else, and a different outcome might result, but, given this different combination of factors, there still will be just the one possible outcome.

            You can imagine as many different combinations of individuals, histories, knowledge, observations, or whatever, and imagine various ranges and patterns of outcomes you might predict would result. When you claim you’re exercising your “free will,” you’re typically engaging in exactly that sort of imaginary fantasy.

            But those imagined scenarios no more exist than does Peter Pan’s Neverland.

            In the real world, exactly one scenario arises, and the function dictates the one-and-only outcome.

            b&

          4. In all honesty, Ben, it seems to me that what you say reduces itself to a banality. Yes, in the particular circumstance ‘a’, with factors (conscious & unconscious) ‘xyz’ influencing the outcome, such and such a decision was taken. That is all that you are saying, and like Lady Macbeth we can say, if we wish, ‘Things without all remedy/ Should be without regard: what’s done is done.’ (Or at least the last part of it – since neither Lady Macbeth or Macbeth are able to have no regard for what they have done: they are consumed with guilt and fear as a result, and that is their tragedy – something that makes them more than those who are able to put such things behind them.) But you have surely had the experience of taking a difficult decision and then subsequently becoming aware for the first time of a factor which, had you known it in time, would have have changed that decision, or becoming aware that you did not attach sufficient importance to a factor which you did take into account, and that had you given that factor its true importance, again the decision would have been different. This does not of course entail that in the particular circumstances in which the decision was taken the decision could have been otherwise than what it was. You have surely had the experience, too, of having decided to act in a particular situation in a particular way that afterwards you regretted, and of being influenced by that regret so that you decide not to act in the same way when you find yourself in a similar situation in the future. and of course that regret will be a factor that determines your decision in that similar situation… You really are not saying so much as you suppose you are.
            I might add that it is surely a virtue to be able to regret what you have done in some particular case and to use the knowledge that you have done ill to make better decisions in the future, and not merely to go along with Lady Macbeth’s dictum that ‘Things without all remedy/ Should be without regard’ – a dictum that might have been Hitler’s or Stalin’s or Papa Doc’s, had they had the moral flexibility and humanity to have been able to entertain it (since it of course implies its opposite). We think better of Albert Speer because he was able to feel regret for what he had done – a priest at the Spandau prison said that he had never come across anybody so consumed with anguish for what he had allowed and perpetrated.

          5. If my position seems banal and limited, it is because the “Free Will” position is a supernatural and exciting one that adds far more onto reality than what’s really there.

            Of course I’ve had regrets, but those regrets are irrelevant to the decision I made at the time. Yes, of course, if I knew then what I know now I wouldn’t make the same decision.

            But, equally of course, if I had the omniscience of the gods, I would have made a different decision, as well.

            In reality, none of that extra knowledge is ever available to anybody, and they’re left to make the decisions they do with the information they actually have. And, given that information and their natures and the rest, they’ve only one choice: the one they actually make.

            It can be very useful to critique the decisions and attempt, after the fact, to figure out if a better decision could have been made; doing so is likely to lead to better decisions in the future. But the fact that we know now that there was something better that could have been done in the past had we known then what we know now…does not in any way magically transfer that knowledge back in time in a way that made that decision possible at the time. At the time, there truly was no (other) choice.

            The third-person omniscient voice makes for a great narrator of fiction, but compatibilists implicitly make the mistrake of assuming that such a privileged observer actually exists.

            b&

          6. Tim Harris,
            “Things without all remedy/ SHOULD be without regard” [My Caps.]
            Lady Macbeth tries to repress their regret and guilt since past actions cannot be changed. The operative word is “should”.

            “..regret will be a factor that determines your decision in that similar [NB.but other!] situation… You really are not saying so much as you suppose.”

            No, you are saying less than you suppose. New situation, new circumstances, =different determined action: in the future Lady Macbeth would be less likely to accept brutal murder as a choice of action.
            Feelings of regret are not willed; they are determined, -do not arise out of, or entail, freewill.

          7. I don’t know where this reply is going to end up – it is to Mogguy, but when I press reply on the advisory e-mail, I am directed not to his response to what I wrote but to another comment in which he or she (probably the former – ‘Moggy’ + ‘Guy’: tom-cat?); so I’m hoping that this will appear in moderate proximity to Mogguy’s response.

            I won’t tell Mogguy to teach his grandmother to suck eggs (that would be rude), but suggest that he should read, very carefully, what I have written, and he SHOULD discover that I have said precisely what he says. I am well able to distinguish between the meanings of ‘similar’ and ‘other’, and nowhere suggest that they are synonymous. ‘New situation, new circumstances, =different determined action,’ announces Mogguy proudly as if he has made a discovery and made a splendid refutation! Could he show me where in what I write I deny this? I think that will do.

          8. And, thank you, Mogguy, but I understand very well what Lady Macbeth meant and do not need you to correct that understanding. Again: read what I write carefully.

          9. Rather than click on the big blue “Reply” button at the bottom of the email, you can simply send an email in reply to the one you wish to reply to. Type your comments (including, if you wish, HTML) at the top of the email and they’ll automatically get added to the Web page and emailed out to subscribers.

            b&

          10. Am trying that. Thanks, although I still think, in Posts with many long threads, to navigate around in them is slow and awkward and the column widths soon become absurd. If they were in strict date order at least that would help find replies. When replying to a post at some distance away chronologically simply quote its date to fix its locality. Arthur

          11. Oh, Mogguy, there’s no need for quite such an apology. I got a bit sniffy and bullying, I’m afraid.

            That, Ben, the extra knowledge that might have led to a different decision was not available at the time of that decision is precisely a point that I was making. Just as it was inplicitly clear in what I wrote, I should have thought, that, no, knowledge subsequently learned cannot be transferred ‘magically’ back in time to change a decision, much as we might want it to do. Nor, despite having read many novels both with and without omniscient narrators, have I ever supposed that I am in the same position with respect to my and other lives as an omniscient narrator, and I doubt very much that anybody else makes that mistake. ‘No man ever steps in the same river twice, for it’s not the same river and he’s not the same man.’ It was all said long ago, and better than most of us manage. Still, I wonder at the fruitfulness of spending one’s time on decisions isolated in some kind of inert logical medium and not attending to how decisions work in the course of our lives, and shape them.

          12. Ben,
            I messed up again. I didn’t notice that my parenthetical “for-anyone” was horribly unclear, but it was.

            Here’s my fourth try. Do the following two sentences at least roughly represent your view:
            “Paul-does-X-at-time-t is the only possible outcome. Moreover, this fact about possibility, unlike for example length in relativity theory, is not relative to an observer, to an inertial reference frame, or to anything else; any other outcome is just impossible, period.”

            Where again, in the ice cream story example, X would be my ordering chocolate, and t would be that first night. And, because your story is close enough to a real life thing that I actually experienced, let’s stipulate that we’re talking about that actual, concrete part of history. Which happened in the actual universe, which we both agree is deterministic.

          13. Ben,
            Cool. So, I propose that possibility is relative to an observer, and specifically, relative to the observer’s place in the causal network. The possibility of an event, measured from one place in the causal network, does not have to equal the possibility measured from another place. This doesn’t make possibility subjective, any more than length is.

            An observer entertains various representations R1, R2, … Rn, which are about events E1, E2, … En. The R’s may be hypotheses, supposed facts, or supposed options. These could be about the past, present, or future, and they could be about others, himself, or inanimate things. And he typically will endorse some representations, and reject others. I use the word “endorse” because it is deliberately ambiguous between acceptance as factual, or selection as the best option.

            Some of those representations, usually more than one, are sufficiently self-referential and related to the observer’s own agency, that his endorsement of representation Ri causes the corresponding event Ei to happen. In other words, Ei is what I’m calling a genuine option for him. To endorse Ri is to choose it (assuming the person grasps the fact of his own agency). For other kinds of representations, like about the actions of others, to endorse those is typically just to believe them, rather than choose them.

            This causal power, endorsing-Ri-causes-Ei, consists in neural structures of his brain, strengths in his muscles, and so on. The causal power exists even when he’s not exercising it, and can exist even if he never exercises it. This isn’t mysterious or spooky, any more than the fact that smoking causes cancer is mysterious or spooky when you don’t smoke. It just consists in certain vulnerabilities of your lungs, to the chemicals in tobacco smoke.

            The ability to bring about Ei, for an unchosen representation Ri, does not just exist in an imaginary universe. Ei only exists in an imaginary universe, but the ability, which is just a set of neural and muscular conditions, exists in this one.

            What good is an option that you will never use? That should be obvious – we seldom know which options will turn out useful, until the last minute. (Or until after, but, oh well.) Best to keep lots of options open.

            If E1 and E2 are genuine options for you, they are both possible-to-you. But that doesn’t mean they are both possible-to-me. My endorsing R1 (the representation of E1) won’t bring about E1, where E1 describes some future action of yours. This doesn’t make possibility subjective. We can each identify the other’s position in the causal network, and thus infer what is or isn’t possible as measured from the other’s location.

            If something X were impossible for you, that would imply that there is no self-fulfilling prophecy “I will do X!” for you. But a genuine option is precisely that, a self-fulfilling prophecy. There is a real-world causal relation between your attitudes toward X and whether X will happen. This can still be true even if X is something you never endorse.

          14. his endorsement of representation Ri causes the corresponding event Ei to happen

            Yes…and no. Entirely no, in fact.

            The boulder that crashes into the tree at the bottom of the hill knocks the tree down, but it was the grain of sand that hit the pebble that hit the rock…that hit the boulder that crashed into the tree that knocked the tree down.

            You’re latching onto the proximate cause of an action, but that proximate cause is, itself, as mercilessly subject to causality and beyond your ability to control as anything else.

            I may prefer chocolate over vanilla, but I had no choice but to prefer chocolate over vanilla. And if my preference changes, it’ll be because of factors over which I have no more control over than those that caused me to have the preferences I do today in the first place.

            In other words, just as I have no choice but to pick the chocolate because I prefer chocolate, I have no choice but to prefer chocolate in the first place. And I have no choice but to have no choice but to prefer chocolate, and so on as far up and down the causality and complexity chain as you might care to do.

            That’s the problem. You can’t draw a line on the continuum and say, “Left of here is free; to the right is fixed.” No matter where you look on the continuum, it’s fixed. There aren’t any places where it’s free, and so there isn’t any transition from fixed to free. It’s all just fixed, with different levels of complexity that sometimes — often, even — make it difficult to figure out what’s banging against what.

            Don’t mistrake “I don’t know” with “therefore free.”

            b&

          15. Ben,
            I do latch on to proximate causes. This isn’t to deny the rest of the causal network, but proximate causes are especially important. It’s largely by identifying them and manipulating them that humans have become so successful. On a site called Why Evolution Is True, this seems like an important point to keep touching on.

            The continuum isn’t fixed. Relative to any observer-actor, parts of it are up to them. This isn’t merely reflective of lack of knowledge – when something is up to me, my trying to find out what I will do as if there were some independent fact of the matter beyond my endorsement, would be a mistake. When you imply that people infer they’re free because “I don’t know”, you get it exactly backwards. It’s because they are free, that they can’t know until they decide.

            Suppose you’re a reporter for Gallup, and they sent you to forecast the elections in Bandwagonia. In Bandwagonia, people vote for whoever they think is going to win, and they take the Gallup poll extremely seriously. If you forecast that Party A will win, Party A will win. If you forecast B, B will win. You are aware of all this. In light of this evidence, which conclusion are you forced to draw? That Party A will win? Or Party B?

            Neither, of course.

            When you declare your own future action, you are playing the roles of both the Gallup reporter, and the only voter. You can’t “know” (if this means “discover”) what will happen. You can only decide.

          16. Paul, may I reply as a dialogue using some statements you made in yours to Ben: You: “The continuum isn’t fixed.” Me: “The continuum comes in two discrete parts separated by the present instant. The known past part-continuum is fixed. The future part-continuum is determined but unknown.”

            You: “Relative to any observer-actor, parts of it are up to them.” Me: “Relative to any observer-actor, parts of the future continuum are up to him.”

            You: “when something is up to me, my trying to find out what I will do as if there were some independent fact of the matter beyond my endorsement, would be a mistake.” Me: “When something is up to me, my trying to forecast what I *should* do as if there were some independent fact of the matter beyond my endorsement, is rational.”

            You: “It’s because they are free, that they can’t know until they decide.” Me: “They can’t know until they decide because the decision is in their future part-continuum: the decision itself is an effect arising from all the prior circumstances. How does this make them free?”

            And similarly: You: “You can’t “know” (if this means “discover”) what will happen. You can only decide.” Me: “Agreed, but any decision you do make is a causally-determined effect.”

          17. Paul, still, all you’re doing is throwing in enough layers of obfuscation (including recursion) that you can’t follow the bouncing ball any more. But the instant you start peeling away those layers, it becomes obvious that it really is just one giant Rube Goldberg contraption that’s guaranteed to do this-and-such whenever the ping pong ball hits the rubber ducky at just the right speed and angle.

            b&

          18. Ben,
            It’s not about recursion. Recursion takes the output of step i, and uses that as input to step i+1. What a rational agent knows about herself is different: she comprehends a set of causal input-output relations, all pertaining to the current state of affairs, and uses that information. She knows that if she endorses representation R1, that causes action A1, while endorsing R2 causes A2. This frees her from the need to predict where the bouncing ball leads, and sing her own tune. Her own tune will be exactly where the “bouncing ball” goes, and it will go there because she made it go there. The process of decision is deterministic. It is not constraining.

          19. Your example of a trusted poll changing the outcome of an election is a perfect demonstration of recursion.

            And, once again, your example here is just of a Rube Goldberg contraption where the output is perfectly determined by the input but in an obfuscated way.

            b&

            >

          20. Ben,

            The recursion in the Gallup poll example happens after the reporter makes his decision/prediction. I’m focusing here on the time during which the decision is made, because that’s where y’all incompatibilists think the problem lies. (Or that, plus earlier times.) The important concept is not recursion, but self-reference. It’s the self-referential nature of the reporter’s verdict on the election which frees him from treating it as a fact to be learned, and allows him to treat it as a decision.

            The recursion does help to create the self-reference, so now I see why you bring it up. But the self-reference is key.

          21. Self-reference is recursion.

            And there’s nothing “free” at all about it. If you think self-reference / recursion is the “key” to “freedom,” you can stop right there. Though it can seem magical and weird, especially to novice programmers, and though it can be a particularly powerful tool…it’s every bit as deterministic as anything else. Look behind the curtain; no magic there.

            b&

          22. If, Ben, you could get away from the simplicities of your preference for chocolate over vanilla ice-cream, or vanilla over chocolate – I can’t remember which you prefer and am not greatly interested, just as I am not greatly interested in your sexual preferences – and address… oh, say, the matter of a general or admiral addressing the question of how to win a battle, or a scientist addressing the question of how to explain some complex phenomenon, or an artist of some kind seeking to solve some difficult artistic (architectural, musical or literary) problem, or an engineer addressing some difficult problem in his or her field, or a mathematician seeking to prove something ; if you would do this, and throw some genuine light on the matter, then I would have rather greater respect for what you say than I am able to manage for the pat little certainties that seem to be your stock in trade. I am sorry if this is discourteous, but I find your air of absolute certainty about difficult and important matters (particularly when you are preaching to those who have succumbed to that air instead of attending to what is actually being said by you) to be at times not altogether warranted – to put it mildly.

          23. Tim, the reason I’m taking a reductionist approach is because that’s how rational analysis works.

            I’m a professional trumpet player, one of the many who doesn’t make a living at it in the area. The day job is database (and related) programming. And there’s no more choice in what I do in either of those settings than when I decide on a flavor of ice cream.

            It’s very obvious in programming. Yes, as the phrase goes, “There’s More Than One Way To Do It.” But you get two broad categories of “other ways”: ones which are functionally identical and ones which are logically identical but not functionally identical.

            The former are vanilla v chocolate matters. For example, a good compiler is going to create the exact same binary code from either of these snippets:

            int i = 0; while (i < 10) { printf(“i = {0}”, i); i = i + 1; }

            for (int i = 0; i < 10; i++) { printf(“i = {0}”, i); }

            Which you choose is going to be mostly a matter of personal style. And, in cases where there’s some overriding factor…that’s just like the example where you were disappointed with your last experience but your friend couldn’t stop raving about hers.

            The other option is where the end result is logically the same but there’s a significant implementation difference. Generally, you’ll have one way that’s quick and easy to write but that is slow to execute and another way that’s quite fast but requires a lot more work to code. And, again, you have no choice. If the quick-and-easy way is “good enough,” you go with that until it becomes a problem, and then you switch to the fast-but-difficult way.

            And a similar type of calculus applies at pretty much all phases of an application’s lifecycle. It all gets broken down into a series of, “Give us such-and-such,” and either the constraints are such that there’s only one way to do it at all, or your “freedom” is the same as picking ice cream flavors — a “freedom” I hope we’ve already sufficiently determined is illusory.

            You can extend that to literally any other scenario you like. You’re building an house? Well, here’s the blueprints you’re supposed to follow. You can deviate from them, but you’ll get fired or sued. Maybe you’re a person of low moral character and you do so anyway, but you still didn’t have any choice in the matter.

            It should be clear by now that the complexity of the various real-world scenarios you refer to only obfuscates the decision-making process such that it’s less readily apparent that it’s an inevitable chain reaction. But, again, that’s true of every other phenomenon we ever investigate in science. Once you break it down to its constituent elements, the picture becomes clear…and, in particular, the illusion of freedom vanishes.

            “Freedom” is, in this context, is ultimately just another word for “It’s too complicated for me to follow all the bouncing balls.”

            Cheers,

            b&

      2. Ben,
        There’s no oxymoron. That is a giant charitable interpretation failure on your part. It ignores the way people actually use the term (outside of theology seminar, at least).

      3. Hi Paul,
        I,too found the other post closed!
        Thanks. Your feelings are reciprocated. Both gear-heads maybe though obviously different cog-nisantly and separated by at least one generation (my elder Son is 70 next month, -I’m 94). Sorry, the Jenann Ismael video link won’t work on my 4year-old Mk1 iPad. Had a quick look at the beginning of her book “The Situated Self” and barely understood any of it.

        JAC says above:
        “I don’t believe in “moral accountability,” which to me implies that one can choose freely. (Indeed, the survey of Sarkissian et al. shows that many people connect a belief in determinism with an absence of moral responsibility.)
        Later
        My problem with “morality” is that it’s deeply connected with the notion that someone could freely have chosen to be “moral” versus “immoral””.

        Of course, every decision ponders choices. At most only one of these choices will happen, also this choice-process must be doing something very good for us, from an evolutionary point of view. But this is “freewill-neutral” and equally true for Determinism
        Many people equate choice with freewill because that’s what freewill means -to THEM, -but that is no justification of it. Many people equate God with natural existence because that’s what God means -to THEM. Doesn’t make it rational belief!

        There are two alternatives in this discussion: (a) We have freewill or (b) We have no freewill.

        Compatibilists hold that my opinion of “no free choice” is a FREE selection. What would change my opinion?
        I think the Compatibilist’s idea of “free choice” is a DETERMINED selection. What would change him into an Incompatibilist?

        I say it would take PHYSICAL change to the respective brains and/or their contents (thoughts). But, of course, Compatibilists will say that the physical change in the composition of our brains will be made by the agency of our freewill -which brings us right back to square 1.

        So, why is this evident interest of eminent of physicists and cosmologists in the freewill debate so common -and heated. Physicists and cosmologists are also as much interdependent humans as anyone else and recognise that “good” behaviour is essential to all humans in our communal way of life. Everyone (except me?) seems to think that the paramount factor in assessing personal responsibility for human behaviour is HOW we make choices, whether the CONSCIOUS decision-making process free or determined.

        I think (unusually?) that the evolutionary fitness of a CONSCIOUS action denotes the fitness of the body/brain that makes it and, to me, this evolutionary fitness is the scale of the behaviour’s moral worth, -NOT “the process” making the decision.

        For example, ignoring the process of HOW he chose to do it, when one human CHOOSES to kill another human then his body/brain is physically such that he COULD and DID decide to kill in the known pertinent conditions that existed for him and his victim at that time. It indicates, to me, the standard of his moral responsibility.

        ‘Moral’ is a useful word, why refuse to use it just because of its historic connections with religious irrationality? Religion uses other words such as mercy, compassion, love, honesty, etc. All of which the newest of atheist’s will include in his vocabulary. Morality should have a respectable rational meaning.

        1. Hi mogguy,
          “The Situated Self” is a great philosophy book, but it’s a challenging read, indeed.

          Most compatibilists think that your choices are both free and determined.

          You compare the equation of freewill with choice, to some people’s equation of “God” with the natural universe. But the situations are different.

          Some people use “God” to refer to the universe as a whole. But these people tend to be ones who believe that the universe as a whole is self-aware, and is purposefully evolving in the way that it does. I think if that were true, that really would count as a god. And I think that if you asked them, what if the universe weren’t self-aware or purposeful, they would say, “well in that case I wouldn’t call it God.”

          Contrast some recent experiments on what people take “free will” to mean:

          Nahmias and his team told 278 participants a story of a future neuroimaging technology that allows perfect prediction of decisions based on a person’s brain activity, recorded by a special skull cap. In this future world, a woman called Jill is fitted with a skull cap that allows scientists to predict everything she’ll do with 100 per cent accuracy, including how she’ll vote in upcoming elections. Contrary to expectations, 92 per cent of participants said that Jill’s voting decision was of her own free will.

          In another version of story, the scientists didn’t just predict which way Jill would vote – they also manipulated her choice via the skull cap. In that scenario, most participants said that Jill did not vote of her own free will.

          When told (in effect) that Jill’s decision making is a deterministic process, the survey participants said that’s still free will. But when her internal choice process was pre-empted and interfered with, they said it wasn’t. The determining factor (pardon the pun) for their judgments about whether the term “free will” applies, seems to be whether the choice was hers.

          Of course, I suppose someone could still insist that no, what free will means is a contra-causal supernatural soul, and 92% of people just don’t know what free will means. But I think the shoe is on the other foot.

          1. Most compatibilists think that your choices are both free and determined.

            Thus the objection from incompatibilists: “free will” is the “freedom” that is not free. You are free to choose anything you like, so long as you make the one-and-only choice which circumstances have caused you to choose.

            b&

          2. Paul, “92% of (278) people just don’t know what free will means” True: the normal perception of Freewill and Choice is that these words have a common meaning, are interchangeable. As you say: “The determining factor (pardon the pun) for their judgments about whether the term “free will” applies, seems to be whether the choice was hers.”

            Why is this such a common conception? (1) Without alternatives of action to choose from, one is completely restricted to one single decision: one has NO freedom of choice, i.e. not free will. (2) With alternatives of action, one is NOT restricted to any one action, one HAS freedom of choice to decide: this is equated to the opposite of no freedom of choice and therefore is equated to the opposite of not free will, i.e. free will. If alternatives of choice exist then free will exists -in everyday usage.

            However. No-one else could be Picasso. Only he possessed the abilities necessary to paint that way. Certainly, Picasso could have chosen not to produce multi-million-dollar paintings but only if his genes, his lived experiences, the time and the area where he was born and matured, his health, and all manner of circumstances had been different. Of course there were always possible alternatives from which he had to choose. But, being that unique human Picasso…

            Determinism says could not have decided otherwise than choose the way he did.

            But, in opining that he was “determined” (could not help but) to be this extra-ordinary human-being, should not prejudice, let alone cancel, the fame, admiration and respect accorded to his genius and achievements. What applies to Picasso applies to everyone: the way they are viewed and the way they are treated by communally-interdependent society, -that is, just as it would be if all our conscious decisions WERE by Freewill.

          3. mogguy,
            You say that Determinism says could not have decided otherwise than choose the way he did. But, that’s a key issue of controversy. I contend that ability to do otherwise does not imply any probability to do otherwise. Determinism says that you won’t take the other option, not that you can’t. The fact that there’s one option you will take, and another that you won’t, is no big deal – it’s just a deepity.

          4. Paul, You say:“Determinism says that you won’t take the other option, not that you can’t.” No, I won’t accept this. Determinism says that you CAN’T take the other option. There is only the ONE possible outcome! We do not behave as trains running to signals on rail tracks (save for knee-jerk reactions) but more like ships out in mid-ocean using all available information to decide on the best direction to steer our course.

            To Ben you said “What good is an option that you will never use?” BUT the person faced with the problem of choosing one option from two or more has no idea whether an option is determined or not. He is not (consciously) constrained to take any particular one of them; his choice to the best of his (conscious) knowledge seems, to him, a free one. But, nevertheless, those constraints are there in all the physical circumstances and conditions in which the chooser is embedded.

            When I am faced with a choice I agree I will choose my option by the same process as if I am a dedicated compatibilist, the fact that I know my choice is determined doesn’t help or influence the process of decision-making at all, -though perhaps, hardly at all is more correct to be honest.

            Which brings me back to the last and, to me the important, part of my previous reply starting at “However” where I used Picasso (who happened to be big news yesterday) to try to advance a different way of thinking about Determinism v. Freewill and their consequent attitudes and effects on human behaviour and morality.

          5. Determinism says that you CAN’T take the other option. [not just that you won’t]

            How do you figure? What is the logical argument for this alleged implication?

            I silently agreed that in scenarios where Picasso hadn’t painted as he did, his genes or experiences would have been different. Its truth was obvious, but its relevance dubious, so I didn’t comment on it. Facts about Picasso’s actual genes and experiences are all contingent (defined in this Wiki), even if we throw in all the laws of nature as axioms. So you can’t deduce conclusions of absolute necessity from those facts.

          6. Hi Paul, “Determinism says that you won’t take the other option, not that you can’t” So despite saying that you WILL (no alternative) take this option, Determinism, nevertheless, also allows you the contradictory premise that you CAN take another option. When, where and how, I ask?

            “Facts about Picasso’s actual genes and experiences are all contingent,” [not necessarily true] ”even if we throw in all the laws of nature as axioms. Since Picasso is dead and his genes and experiences are in the unalterable past, how are the actual events of his life, as it we know it, NOT the reliably evidential truth? There is no alternative to the observed past: that is, no scenarios in which Picasso was different or could have chosen to behave differently.

            Past events are unalterable. Picasso’s existence was in the past. The events of his existence are unalterable. The “events of his existence” include his actual genes and experiences. Is that illogical? l The “Absolute” of anything is an unattainable idea outside religious or similar hypotheses. We only know varying degrees of Truth which never will/can reach the Absolute of Certainty. (IMO anyway!)

          7. mogguy,Saying that you will do something doesn’t imply “no alternative”; it’s just a prediction.  A prediction might not specify whether you, or something else, causes the event.  If you are the cause, then it’s up to you.  The predictor isn’t in control, you are. There is nothing you or I could do now about Picasso’s genes or environment.  But there are plenty of things Picasso could have done while he was alive which are such that, if he did them, his genes or early environment would have been different.  That just follows from CPT invariance of the fundamental physical laws, which sets up a 1:1 correspondence from the present to the past (as well as vice-versa).  The whole past isn’t “fixed”, just the thermodynamically irreversible parts, and even those parts are “merely” a matter of statistical probability.  My next sentence is hard to parse, but I don’t know how else to say it.  When the past circumstances we are interested in are whatever happens to correspond to our exercising various options in the present, then those probabilistic considerations are bypassed.  The deck is stacked, in our favor.  We can do whatever we want.

            (See my comments to Ben for a definition of “option” that’s consistent with determinism.)

          8. Paul,

            “The whole past isn’t “fixed”, just the thermodynamically irreversible parts, and even those parts are “merely” a matter of statistical probability.”

            Can you give an instance where any part of the whole past has changed and so justify this (to me) strange irrational claim? That is: that such and such an event became a different one on such and such a date.

          9. mogguy,
            That’s not what I’m saying. You can’t change a given past event, but then, you can’t change a given future event either. Take “Paul Torek will be celebrating New Year’s on 1/1/2016 at 12:01 am”. I can’t change that event. I can make it true, or make it false, but I can’t make it first be true and then be false! If that statement is true, it’s true now, it will be true at 1/1/2016 12:02 am, and 12:03 am, and so on.

          10. Paul, You say: ”you can’t change a given past event” this event being one within the “the thermodynamically irreversible parts”.

            This implies that there are un-given past events that could produce reversibility. Since there is only one Past where do these unfixed, un-given past events happen, how many are there, a countless number maybe, and how do we influence them?

            You also say: ”you can’t change a given future event”

            Let’s make “Paul Torek will be celebrating New Year’s on 1/1/2016 at 12:01 am” a given future event and is therefore, by your definition, unchangeable. It is determined that you will be fit, ready and able and have the wherewithal to celebrate.

            You say: ”I can’t change that (given future) event ” Ergo: you cannot make it false because it is (your words) a given event and must be true.

            How do you explain the following: “I can’t change that event.” (yet also) “I can make it true, or make it false, but I can’t make it first be true and then be false! If” (as we have taken it to be) “that statement is true, it’s true now, it will be true at 1/1/2016 12:02 am, and 12:03 am.”

            I am at a loss to understand what “but I can’t make it first be true and then be false” means with respect to your given event which, by definition, is inevitable!

            Again to Ben you say: “The process of decision is deterministic. It is not constraining.” I thought a constraint was a limiting factor: the deterministic decision-making process is limited to one-only actual decision therefore must constrain (make un-free) your decision. The nuance you claim escapes me!

          11. mogguy,
            “Given” in my discussion just meant “the particular event we are referring to”, it doesn’t mean handed down by God. Suppose you now predict “Paul will party on New Year’s” and as it later turns out, I do. Then your prediction referred to that event, and was correct. This works regardless of determinism, free will, etc.

            If I said “a given past event”, that likewise just meant we are referring to a particular past event.

            Maybe I’m being too persnickety, but you can’t literally change any event, past or future. Because change means “first being one way, and then another”, but whichever event we refer to, it only happens once, and it is whatever way it is. Of course, when people say “Steve Jobs changed the future” they don’t mean that the future was literally first one way, and then became another way. They mean that it would have gone differently without him. But if we’re gonna do philosophy of time, we need to be damn careful not to talk that loosely.

            You can’t change any past event, thermodynamically reversible or not. But, for a temporally-neutral definition of “causation”, such as that given in Pearl’s Causality, you can cause some past events. On the other hand if you define causation as a strictly past-to-future kinda of thing, then of course we can’t “cause” anything in the past, but don’t mistake your tautology for a profound insight.

          12. I contend that ability to do otherwise does not imply any probability to do otherwise.

            You’re proposing a contradiction.

            Might as well suggest that there’s an ability to fairly flip a fair coin millions of times in a row and have it come up the same side every time without implying any probability to do otherwise.

            And, in this instance, the case is even stronger. You could similarly claim that there’s an ability to calculate a “4” in the decimal expansion of “1/3” without implying any probability that it’d actually happen.

            The two statements are equivalent: we know that there are no “4”s in the decimal expansion of “1/3” because it’s impossible for there to be one; and we know that it’s impossible for there to be any “4”s in the decimal expansion of “1/3” because there aren’t any “4”s actually in there.

            The proper term for something that can’t actually happen for whatever reason is, “impossible.” Doesn’t matter what it is that keeps it from happening, and it could well happen under other circumstances; for the phenomenon under question, if “never happens” is an accurate description, “impossible” is the correct label.

            b&

          13. Ben,
            Select a real number at random from the interval [0,1). Whatever number you get, call it r, there was a 0 probability of getting that particular number. But it was possible.

            So much for your alleged contradiction.

          14. Paul, if you think that any random number generators can actually generate irrational numbers other than by picking their representative symbols out of an hat, you don’t know how computers work and don’t have a good grasp on the fundamentals of computer science.

            An actual representation of an irrational number would require infinite memory, something that doesn’t exist.

            This may actually point to some of the conceptual difficulties you’re having…there’re all sorts of mathematical constructs that are quite useful in all sorts of settings that don’t, when it comes right down to it, actually represent reality. And that’s been the case right from the get-go…you can draw representations of geometric figures on a piece of paper, but no actual geometric figures actually exist in the real world, merely things (such as your drawing) that superficially resemble them to one degree or another.

            A real triangle? It’s as imaginary as your choice of whether or not to draw it.

            b&

          15. Ben,
            However merely-abstract mathematical objects may be, mathematicians do know their logic. The foremost imaginary object around here is your alleged contradiction between possibility and zero-probability.

          16. Paul, the equation between “never happens” and “impossible” is as fundamental as it gets. The Sun never rises in the West; it is impossible for the Sun to rise in the West. Nobody has ever flipped a fair coin ten thousand times in a row; it is impossible to flip a fair coin ten thousand times in a row. Nobody has ever (correctly) calculated a 4 in the decimal expansion of 1/3; it is impossible to calculate a 4 in the decimal expansion of 1/3.

            If something literally never does or ever will happen, it’s truly impossible for it to happen.

            b&

            >

          17. Ben,
            We are talking about particular human actions at particular times and circumstances, each of which happens either one time or never. Adam didn’t do A just now: could he have? You cannot infer from proposition “not A” to “not (possibly, A)”. That’s just bad modal logic.

            If there really were a contradiction in what I’m saying, you ought to be able to demonstrate it in modal logic.

          18. This is a scientific observation. Logic must always be second to science; logic, for example, with its non-contradiction principle would have us believe that a photon must either be a particle or a wave, and so on; basically everything “weird” that happens in Quantum and Relativistic mechanics violates some logical principle held dear since the time of the ancient Greeks.

            And the observation is trivial. As surely as the Sun always rises in the East, human actions are the result of an inevitable cascade of events. The cascade is much more complex than that of an avalanche and therefore that much more difficult to predict — and we can’t even predict the avalanche save at gross levels.

            But we know that neither the rocks in the avalanche nor the avalanche itself have anything resembling “choice” in the course of their fall…and we know the exact same is true of humans.

            You only think you do because a small part of that avalanche includes some recursive computation that analyzes your current situation and compares it with similar patterns…

            …but you still haven’t grasped that even all that recursive computation, all your analysis…it’s still part of the avalanche.

            Until you accept that it’s all part of the avalanche, as long as you keep trying to magically separate your cognition from the physics which comprises it, you’ll be stuck with the quasi-religious fantasy of free will, of the puppet who pulls his own strings.

            b&

          19. Ben,
            I’ll combine my reply to the other comment in here:

            Self-reference is recursion.

            They’re not equivalent. The recursive definition of a factorial only refers to “n!” in the definition of “(n+1)!”: each step is separate, and there is a base case of n=0. This separation of base case from higher cases is what makes it – unlike the reporter’s attempt to calculate (instead of just decide) his Bandwagonia prediction – well-defined. Sure, you can say “the definition of factorial refers to a factorial,” but it doesn’t refer to the self-same step in the chain.

            There’s no magic in the Bandwagonia story. You’re pulling the accusation of magical thinking from nowhere.

            logic, for example, with its non-contradiction principle would have us believe that a photon must either be a particle or a wave, and so on; basically everything “weird” that happens in Quantum and Relativistic mechanics violates some logical principle held dear since the time of the ancient Greeks.

            There’s nothing in logic to dictate that a photon must be a particle or a wave. I certainly agree that modern physics destroys principles that seem intuitive to most people, though. Like “causation is inherently unidirectional”, or “the past is fixed, so everything that flows from it is also fixed,” for examples.

            human actions are the result of an inevitable cascade of events.

            There’s nothing inevitable about it. Evitability is all about the fact that human actions cause certain things that would otherwise happen, not to happen. But all kinds of objects and processes prevent certain things that would otherwise happen. Torrential rains prevent the spread of forest fires. Wolves prevent rabbit populations from soaring. Etc. Incompatibilists apparently believe that everything in the physical universe has causal powers – except human beings! Human beings are special: specially powerless and miserable. And the Universe is a Puppet Master, manipulating its self-conscious parts, like an evil God. That’s the incompatibilist theology.

          20. Self-reference is recursion.

            They’re not equivalent.

            No; self-reference is the defining characteristic of recursion. Consult any introductory programming textbook.

            Incompatibilists apparently believe that everything in the physical universe has causal powers – except human beings!

            You have spectacularly missed the point.

            Incompatibilism is the position that humans have no more and no less causal power than anything else in the universe. There’s nothing special about humans; that’s the whole point. Yes, we happen to be more complex than average — but so do Rube Goldberg contraptions. And it can be difficult to predict human behavior…but it’s also difficult to predict the weather. Neither Rube Goldberg contraptions nor thunderstorms have any more or less choice, any more or less causal power, than an human.

            The compatibilist position claims special powers for humans and other agents. Yet we know from physics that no such powers exist.

            The compatibilist would endow us with a causative soul-like entity with magic powers to act other than according to the inescapable laws of physics that, honestly, for the life of me, I can’t distinguish from the standard issue Christian soul.

            The incompatibilist recognizes that there is a continuum of complexity from thermostat to pocket calculator to smartphone to supercomputer to human, and that all are merely more and more complex variations on the exact same theme, with no fundamental differences at all.

            The compatibilist inserts the “…and then a miracle happens…” step somewhere after the supercomputer and before the human. Just like the Christian, save the compatibilist insists that this causative power still somehow comports with physics…yet, the fact remains that physics has no loopholes that permit an escape from universal causality, despite all compatibilist claims to the contrary.

            b&

          21. Ben,
            StackExchange finds a difference between self-reference and recursion. Commenter Ben Crowell mentions the sort of self-reference I’m highlighting:

            What about this statement: “This sentence has five words.”

            The Gallup reporter’s self-reference is degenerately circular like that, with no base step and higher steps. Doing his causal calculations, he derives the result “This prediction will be correct if it matches this prediction,” or in short, he has the equation X=X. The solution is unforced, and not because of his ignorance, but because of his knowledge: knowledge of his own causal role.

            That doesn’t mean his decision is uncaused. It’s caused by him. By a real, physical, flesh and blood object called a human being. With beliefs and desires that motivate (which is causative, but is not coercive) decisions via reasoning. Where is the nonphysical soul in this story?

            You’re on a witch hunt, but there is no sorcery here. The only spooky business is calling the universe a Puppet Master, as if it were a cosmic Agent, Who manipulates people. That might seem to exonerate people from whatever they do, and take their agency away, but the cosmic Puppet Master is unreal. The only agents are us.

          22. “This sentence has five words.”

            Before we continue, let’s stop for a moment and consider this sentence. What does it say? Well, it starts by referring to a sentence — but not just any sentence; one sentence in particular: a sentence that reads:

            “This sentence has five words.”

            Before we continue, let’s stop for a moment and consider this sentence. What does it say? Well, it starts by referring to a sentence — but not just any sentence; one sentence in particular: a sentence that reads:

            “This sentence has five words.”

            Before we continue, let’s stop for a moment and consider this sentence. What does it say? Well, it starts by referring to a sentence — but not just any sentence; one sentence in particular…

            …and by now I hope you get the point. Self-reference is recursion.

            StackExchange

            For the record, sites such as StackExchange are infamous amongst programmers as being resources of last resort, suitable only for desperation or the lazily incompetent. Proper context for citing them is, “The server kept throwing a Frobnotz error, but there’s no mention of it in any documentation, online nor printed, and Web searches were coming up short. Fortunately, before I had to start decompiling the server binary, I stumbled across some poor schmuck on StackExchange who had the same error. Somebody helped him kludge a workaround, but that told me enough to figure out that it was really caused by an improper type conversion from bad data from the upstream provider, and I was home free after that.” You do yourself no flavors by citing it as any sort of an authority.

            The only spooky business is calling the universe a Puppet Master, as if it were a cosmic Agent, Who manipulates people.

            If there is any agency implied in my language, it is solely an artifact of the dualism that inescapably permeates English. Of course, the Universe is no puppet master; shit just happens, but it happens in as predictable and inevitable way as any other Rube Goldberg contraption.

            Come to think of it, that’s probably the least worst summary of the incompatibilist position.

            Shit happens.

            b&

          23. Ben,
            I still think you have an overbroad definition of “recursion”, but, whatever. By examining its syntax, we can see that the five-word sentence is truth-evaluable, and true. Call it a recursion if you want, but it’s a damn short one, not very comparable to standard recursions like the definition of a factorial.

            shit just happens, but it happens in as predictable … way as any other Rube Goldberg contraption

            I’m on board with that.

            and inevitable

            But not that. It doesn’t follow from “predictable”, and it doesn’t follow from “caused”. It’s just a gratuitous assumption.

          24. Okay. Let’s take it one step at a time.

            Would you agree with me that it is inevitable that the Sun will rise in the East tomorrow?

            b&

          25. Ben,
            Sure. Even if all 7 billion humans intended otherwise, the sun would still rise, so that’s inevitable. It’s only effects that are sensitive to the causal powers of agents, that are evitable.

          26. Ay, but there’s the rub. Keep digging, and you discover that there actually isn’t anything sensitive to the causal powers of agents, unless you wish to invoke supernatural spiritualistic dualism.

            That’s the lesson of physics. Newton figured out that the same laws that causes apples to fall down also keeps the planets in motion. No gods shepherd the planets through the skies; no intentions keep them on their course.

            The same applies to everything, no matter how complex.

            The instant you wish to claim that something doesn’t, in essence, fall down, that has some sort of a choice in the matter…

            …that’s the instant when you reject physics and go off the rails yourself.

            b&

          27. OK, someone feel free to explain exactly where/what our Deterministic “choice” comes into play. Not or rising or setting of the sun, not on vanilla v chocolate ice cream.
            Where IS choice allowed, or did I misread tha we (supposedly) *have* some choice in our actions.

          28. That’s the $64,000 question that’s yet to be answered, save through appeals to Platonism or complexity or unpredictability or randomness or, most commonly, an insistence that different-but-similar situations have different outcomes, so not-different-and-identical situations can therefore also somehow have different outcomes.

            Short version? That illusion of agency, the imaginative power to jump from imagined timeline to imagined timeline, is nigh on impossible for some people to see past. They see that one morning they drank coffee and the other tea, so it’s obvious that they had a choice, and are completely oblivious to the fact that, the morning they drank coffee circumstances were such that coffee was the only remaining option for them at the point they made it, making any other hypothetical choices as imaginary and unreal as something out of an Harry Potter novel.

            b&

          29. Ben,
            Now you’re back to claiming humans are special – specially powerless. Only the neural representations of actions within human brains, among all the features/objects in the universe, lack causal powers. Or maybe we should ignore neuroscience, and say that human intentions are not features of the brain, but rather features of spooky souls, and therefore nonexistent!

            I’ve got a better idea. Let’s take science seriously. We will then see that people deliberate, choose, and act accordingly – all of which just are neural (+muscular, for action) processes amenable to scientific study.

          30. Paul,
            I think the problem is fundmentally that if one is going to adopt the viewpoint of physics, then there is no fundamental difference between a human being and a lump of granite, which makes me wonder how useful – in the sense of explaining anything – such a viewpoint is (even though in the – very distant – end it is doubtless true) – but if we are going to admit that human beings are influenced by such things as cultural upbringing, exhortations, arguments, a sense of honour, the desire to be a person of integrity, a native rebelliousness, the desire to win over others at any cost, the experiences they have, pressures to conform, desires not to conform, fears as to what might happen if one doesn’t conform, then I don’t think that physics is in a position to say very much, as I think Ben at some point conceded (correct me if I’m wrong, Ben).

          31. Tim, my whole point is that “free will” is an incoherent mess with negative explanatory power, seeing that it’s a spectacularly incorrect attempt at a model.

            If you want to explain human behavior, you would be most prudent to run as far away from “free will” as possible and instead focus on how humans actually work, in all their deterministic and complicated glory.

            It’s not my fault that compatibilists attempt to view determinism through the lens of “free will” and come away concluding that there’s not much point to such an exercise.

            Or, “What good is it to say that the Earth is round when, clearly, that just means that everybody in Australia would fall down off the underside of the Earth?”

            b&

          32. There is a distinction between accepting the splendidly religiose conception of free will and recognising that psychological and social factors influence what we, doubtless mistakenly, call our ‘decisions’ – Jerry, for example, thinks that punishment is one such factor, that religious fervour is one, and that following an argument is one… This has nothing to do with the dreadful heresy of compatibilism. No-one sensible is denying (at least I am certainly not) that scientific studies of consciousness or how we ‘make’ decisions are pointless or worthless.

          33. No, I am not. And you are merely being evasive, it seems to me. Please stop imposing your assumptions as to what others are thinking on your interlocutors. The important causes of the recent mess in Baltimore, Ohio, are not those of physics (though doubtless one could if one knew enough – if one were God – reduce them to purely physical workings), but a century and more of institutionalised racialism. To acknowledge that as an important cause is also to acknowledge that it might be possible to change society so that such racialism no longer plays so huge a part as it has been playing. We don’t have to wait for science to tell us this, and we don’t have time to. Why object to this sort of thing, when so far you have made no objection that I can remember to Jerry’s assertion that accepting unconditional determinism entails that we will be in a position to devise punishments for various crimes that will be both fair and effective as deterrents? Do you think otherwise? If so, please say so. What comes across from the intransigent insistence and the purity of thought in which you seem to take such pride is that nothing issues at all from the assumption that ‘incompatibilist’ determinism is true – and that is something with which I would tend to agree. But at the same time you seem to feel that such determinism is an idea of momentous importance – perhaps because it gives you a stick to beat a certain kind of religious person: but that is really a rather trivial reason for thinking it momentous. One would like a bit of an ‘apologia pro vita tua’.

          34. Tim, determinism in no way implies fatalism, a point that Jerry repeatedly makes.

            We are computational devices that perform functions. For each set of inputs and state, there is one and only one corresponding output. That output itself serves as input / state for the next inflection point, and so on.

            The sum of that causal chain to date is such that I abhor institutionalized racism and will do what little I can to end it.

            Fatalism would mean saying, “Meh. It was inevitable, so why bother trying to change anything?” But that conclusion most emphatically does not follow from the premises.

            “Free will” and the decision-making process is entirely irrelevant when it comes to matters such as institutionalized racism — as irrelevant as whether or not souls and gods are real. “Meh. I don’t have a soul and when I die I won’t be with Jesus, so why bother trying to be a good person?”

            It is, indeed, much the same irrelevance as Newtonian Mechanics has to do with navigating a car. “Meh. Inertia says the car will keep going in a straight line unless some force acts upon it, so why bother turning the steering wheel to avoid that brick wall?”

            Indeed, when it comes right down to it, incompatibilism is as irrelevant as free will itself — just as atheism is as irrelevant as theism. The one exists only as an expression that the other is bullshit; if the bullshit wasn’t such an huge distraction in the first place, nobody would bother reacting against it.

            Cheers,

            b&

          35. Here we go round the mulberry bush yet again. I wonder, Ben, if there is not some glitch in your brain that makes you misinterpret – I nearly said ‘willfully misinterpret’ – what others say so that you can have the satisfaction of attacking your own misinterpretations and sounding authoritative? This time, it seems, I am conflating or confusing fatalism with determinism, and Ben can roll up his sleeves and get down to the pleasing task of putting me right, calling in Jerry for assistance. But I am not confusing fatalism with determinism, and have not come within ninety nine leagues of doing so. I am not talking of fatalism at all. What I am questioning are the seeming assumptions that physical causes are the only properly ‘real’ ones, and that to talk of the kinds of influences that we have on one another, through argument, exhortation, upbringing, education, etc, is to give hostages to the fanatical hordes of Free Willies. I am, however, glad about the concession, extorted with difficulty, as to the irrelevance of these considerations to such things as the evils of instituionalised racialism.

          36. But I am not confusing fatalism with determinism, and have not come within ninety nine leagues of doing so.

            Tim, the whole, “They have no choice but to be [fill in the blank] so why bother trying to change them?” bit you launched into is pure fatalism.

            What I am questioning are the seeming assumptions that physical causes are the only properly ‘real’ ones, and that to talk of the kinds of influences that we have on one another, through argument, exhortation, upbringing, education, etc, is to give hostages to the fanatical hordes of Free Willies.

            Well, that’s the heart of the problem right there.

            To you, argument, exhortation, etc., aren’t matters of physics.

            I know you’re going to be upset at my use of the word, “dualism,” but you’ve just given a textbook example of dualism in practice.

            There is no magical other “stuff” of which mental constructs are made.

            Rather than Jerry, I’ll enlist Sastra to my aid on this one. That second sentence of yours I quoted is a perfect fit for her definition of, “supernatural.” I should note that she and I disagree on the proper way to define the term; her definition is more exclusive and focused than mine. But yours is a case that’s a perfect fit for her definition and one that I’d also instantly identify as such.

            Cheers,

            b&

          37. (BG) Tim, the whole, “They have no choice but to be [fill in the blank] so why bother trying to change them?” bit you launched into is pure fatalism.

            (TH)Can you show me exactly where I have asserted that? I have nowhere asserted that. What I have said is that in the case of, for example, Baltimore, Ohio, the primary cause (which no doubt could be reduced to physics if we knew enough) is institutionalized racism, and, recognizing that, we may try to do something about it. Really, Ben, I don’t want to accuse you of dishonesty, but I find it extraordinary the way you foist on others things they patently have not said and then take issue with your misrepresentations. Come on, be responsible.

            (BG) To you, argument, exhortation, etc., aren’t matters of physics

            (TH)Yes, argument, exhortation, etc are at bottom physics. It is via argument and exhortation that physical changes are brought about in one person’s brain (A’s) by another’s, changes that may have the consequence that Person A, as a result of reading Jerry Coyne’s good book ‘Why Evolution is True’, espouses the theory of evolution and turns away from religion.

          38. Tim, here’s what you wrote that is a textbook example of fatalism:

            The important causes of the recent mess in Baltimore, Ohio, are not those of physics (though doubtless one could if one knew enough – if one were God – reduce them to purely physical workings), but a century and more of institutionalised racialism. To acknowledge that as an important cause is also to acknowledge that it might be possible to change society so that such racialism no longer plays so huge a part as it has been playing.

            You explicitly reject physics as being even remotely relevant and claim that only through such a rejection can one hope to effect change.

            Now, it is certainly true that there is no need to model human society down to the subatomic level to understand what’s going on, and that that would be a most inefficient way to go about societal reform.

            But your statement goes far beyond that, and straight to dualistic fatalism.

            …and your post that I’m replying to, once again, goes straight to the dualism:

            It is via argument and exhortation that physical changes are brought about in one person’s brain

            That’s exactly bass-ackwards and umop-apisdn. The physics is primary; we know this through countless observations going back to Newton at the least.

            The aggregate of the lower-level workings can almost always be easier described as if the lower-level stuff is irrelevant…but it is the lower-level interactions that aggregate to higher-level phenomenon, and the higher-level phenomenon we observe. The higher-level phenomena do not dictate the actions of the lower-level phenomenon; the higher-level phenomena are consequences of the lower-level interactions.

            A claim that “argument and exhortation” can bring about physical changes in a brain is exactly what dualism is all about.

            Skyhooks v escalators, as Dan Dennett would put it.

            Skyhooks v escalators.

            b&

          39. I suggest, Ben, that you open a large dictionary and find out what ‘fatalism’ means. To assert that institutionalised racism has been an important cause of the mess in Baltimore is no more fatalistic than to assert that the cause of Trayvon Martin’s death was a shot fired by George Zimmerman, that the English won the battle of Agincourt because of their longbow-equipped archers, or that last Sunday I had a hangover in consequence of drinking too much Gargery stout (which I recommend, but not in the quantities I was induced to drink it on the previous evening). You are just playing with words, and saying nothing of interest. If that’s how you want to do things, well, then get on with it.

          40. Tim, I suggest you do the same. My dictionary reads, “the belief that all events are predetermined and therefore inevitable. a submissive attitude to events, resulting from such a belief.” (emphasis added).

            Or just read Jerry’s usage of the term as well.

            b&

          41. You, Ben, need to explain why you suppose that saying that A caused B represents a ‘submissive attitude to events’ resulting (note that ‘resulting’) from ‘the belief that all events are predetermined and therefore inevitable.’ But, frankly, I am not interested on your answer since it is very clear that you are not arguing in good faith or interested in truth in this connexion, so I think we should call it a day.

          42. Ben,You compared the dualist’s view of brain-based predictions, to the fact that you can hear a heartbeat through a shirt.  On the dualist view a brain/body is just something that a soul wears for a while, like a suit.  But it would be a strange sort of shirt, that doctors could listen to and confidently predict the *future* activity of your heart.  In the Nahmias et al research, subjects are given a story in which scientists predict Jill’s *future* voting decision, by observing her brain.  It’s hard to see how subjects are supposed to view that dualistically – the brain predicts the future activity of the soul? Even if the subjects taking Nahmias’s survey are dualists, I think they will read the story as telling a materialistic tale, where the brain is the decider.  After all, it’s just a hypothetical story.  They can give their opinions on it, without believing that it represents how real human beings work.  And in their opinions, Jill still has free will in that materialistic story – as long as the scientists don’t pre-empt her brain activity, but merely observe it.

          43. But it would be a strange sort of shirt, that doctors could listen to and confidently predict the *future* activity of your heart.

            Why on Earth should that be so? Barometers have been used to confidently predict the weather for ages.

            b&

          44. Ben,Barometer-based weather predictions tend to leave the forecaster all wet – literally.  But what we know about the “soul” (on the assumption that such exists) pales in comparison even to our meager weather-knowledge.

          45. Tim,
            As I guess you yourself are pointing out, there are a lot of non-fundamental (from a physics viewpoint) differences between a human being and a lump of granite. From a biological, anthropological, or psychological viewpoint, those differences are fundamental. One should use the sciences appropriate to explaining the subject at hand.

            Ben,
            Putting aside your circular arguments from “no choice” to “inevitable” and back to “no choice”, the pattern is clear. You’re clinging to the dualistic misdefinition of “choice”. You should stop deferring to the mystics’ definition of that word; there’s nothing spooky about choice. Meanwhile, actual neuroscientists study choice in blithe disregard of the spooky misdefinition – as well they should.

            Incompatibilists claim the mantle of science, but when it comes to the definitions of “free will” and “choice”, they defer to mystics and theologians. It’s as if a biologist accepted the vitalists’ definition of “life”, and concluded, because there is no elan vital, that nothing is really alive.

          46. Paul, I’m using definitions as consistently as possible. In the context of a discussion on, “free will,” theological definitions are the only applicable ones. Why? Ask the proverbial man on the street why evil exists and the very first answer you’ll get will include the words, “free will.” Ask people if we have, “free will,” and the answer will perfectly mirror that person’s spiritual position on the nature of the sou. And so on.

            Now, if you wish to step away from theology, yes, “choice,” and, “decision,” can be very useful terms…

            …but the problem with compatibilists is that they wish to privilege the choices and decisions made by humans over the ones made by other computational entities.

            If humans make choices, so do thermostats. If humans have, “free will,” so do thermostats.

            But most people reject the notion that thermostats have “free will” and make decisions — for the obvious reason that “free will” is the power by which ensouled spirits make decisions.

            Try an exercise: make a list of entities that have “free will” in the compatibilist sense and those that don’t. Now ask a “spiritual but not religious” person to make up a list of entities that have a soul and those that don’t. Compare the two lists…and you’ll find them essentially the same.

            Because you’re both steeped in the same woo….

            Cheers,

            b&

          47. Ben,
            That was a particularly good comment. It explains where you’re coming from.

            “Ask people if we have, ‘free will,’ and the answer will perfectly mirror that person’s spiritual position on the nature of the soul.” That experiment has basically been done and the results don’t cohere well with your prediction. Not to deny that most spiritualists will give spiritual theories of free will, but there’s a distinction between theory versus underlying definition. If you asked ancient Greeks for their theory of pair-bonding, they’d say the god Eros shot people with magic arrows. Yet, erotic love has survived the death of the god Eros, just fine, thank you very much. The theology turned out to be superfluous. This process is already being repeated with the concept of free will, where fields like law (ability of the accused to conform to the law) and medicine (informed consent) are well on their way to adopting compatibilist definitions.

            If compatibilism is taking over in the real world, why do I bother arguing it philosophically? Because people will cling harder to their spiritualism if they think free will depends on it, which leads to many errors and other bad consequences. And simply because incompatibilism is a mistake that facilitates other mistakes.

            “the problem with compatibilists is that they wish to privilege the choices and decisions made by humans over the ones made by other computational entities.” I don’t find anything objectionable about people caring more about humans than other mammals, more about mammals and birds than insects, and so on, with thermostats and even Apple’s Siri falling way down (or off) the list. I’m sure evolutionary psychologists can give good explanations why we care about certain qualities that humans excel at – but whatever the explanation, we do. So of course we have concepts, like consciousness or self-consciousness or desire or cognition, that refer to these qualities that we care about. And sure, there’s a continuum of computational features between a thermostat and a human being, just like there’s a continuum of amounts of light between night and day. There’s still a useful distinction there.

          48. Paul,
            I think the quarrel is not really between incompatibilism & compatibilism, but between the assertions of certain incompatibilists (which remind me, in their intransigence, and in the sort of hubris and arrogance that comes across – arrogance that is attractive to the sort of people who fall over the ‘real men go to Iran’ schtick – of nothing so much as convinced Marxists, Freudians & behaviourists)and an account of human (and other) life that recognises that determinism is correct and at the same time does justice to human life and makes sense of it.

          49. Tim, we’re not denying that humans think, have feelings that motivate our actions, and all the rest.

            The objection is to characterizing humans as having this mystical spark of the soul of “free will” that separates us from everything else — a mystical spark that violently contradicts all we know about physics and is nowhere to be evidenced.

            Newton showed us that the same force that keeps the planets moving also keeps you held to your chair, and simultaneously erased any possibility that the Earth (and therefore humanity) is at the center of everything.

            Everything we’ve learned since then makes plain that we’re no different at any fundamental level from anything else, just a particular arrangement of the same stuff working in the same basic way.

            “Free will” rejects that notion, and infuses humans with special agency to flout the laws of physics that dictate the paths all other objects take. And, sorry, but we should have gotten past that sort of superstition centuries ago.

            b&

          50. Paul, that study isn’t the same as what I proposed…and, indeed, the results are exactly what you’d expect to find from dualists. The body is just the medium through which the soul interacts. So you can observe the body and understand the soul is reflecting…well, yeah. That’s the whole purpose of the body. And you can manipulate the body and remove the person’s free will…again, duh. Chains do the same thing.

            I don’t find anything objectionable about people caring more about humans than other mammals, more about mammals and birds than insects, and so on, with thermostats and even Apple’s Siri falling way down (or off) the list.

            That has very little to do with computation and much more to do with shared genes…and in none of it are questions of “freedom” applicable.

            Besides which, Apple’s Siri is already substantially more sophisticated than many insects….

            b&

          51. Ben,

            You interpreted that experiment this way: “So you can observe the body and understand the soul is reflecting…”. That’s a pretty strained interpretation. Subjects were asked to suppose that Jill’s action, including her voting, was predicted using a brain scan. If her soul were the root of her action, then a brain scan would be inadequate to predict it before the election. Also, the article I linked to left out some experiments. Here’s additional info from the abstract of the research paper:

            In Experiment 2 the scenarios increased the salience of the physicalist implications of neuro-prediction, while in Experiment 3 the scenarios suggested dualism, with perfect prediction by mindreaders. The patterns of results for these two experiments were similar to the results in Experiment 1, suggesting that participants do not understand free will to require specific metaphysical conditions regarding the mind–body relation.

            On caring more about humans than mammals than insects, you write “That has very little to do with computation and much more to do with shared genes.” It’s both – at least, if having desires and cognition and experience are matters of computation – a common view among physicalists, though not the only one. I would be horrified if I thought my descendants would have my genes but would not have desires nor experiences (maybe they are simplistic robots that carry around test tubes full of genetic material – somebody built an AI and told it to preserve humanity, and this is its horrific misinterpretation of that imperative). Genetics explains how we got our desires, but it doesn’t capture the content of those desires – else humans wouldn’t desire to use birth control the way they do.

          52. That, alas, seems to have become Ben’s modus operandi when pressed: evade the issue through blatant misrepresentation and accuse your interlocutor either of saying something he didn’t say, or, more insidiously, of ‘really’ believing, beneath the surface of words which suggest nothing of the kind, in some ridiculous woo. I can’t help being reminded of a certain kind of Freudian, with their little Freudian key that allowed them to reduce any surface phenomenon to an oedipal fixation or whatever. I don’t know why Ben does it. I wish he wouldn’t.

          53. Tim, I’m not.

            I would suggest that you haven’t fully embraced the consequences of physics and still cling to a diluted form of dualism, as evidenced by the quotes from you in my previous email.

            It’s very, very, very difficult to invert one’s thinking. Our experiences are immersed in a most powerful illusion of dualism, and our language itself is inescapably dualistic.

            But it really is really true that thoughts are no more (and no less) real than the wind in a storm. The wind can and will blow you over, yes…but the wind really is nothing more than uncountable numbers of air molecules pressing against each other, and it is that combined kinetic energy that knocks you down. It is most emphatically not the case that the wind moves the air molecules.

            Cheers,

            b&

          54. Paul, you’re approaching the observations from a reasonably rational and scientific perspective.

            But we’re talking about people who think that Jesus died on the Cross for their sins, was Resurrected, and awaits them in Heaven come Judgement Day. In that context, everything you wrote is incomprehensible gobbledygook.

            You already accept, for the most part (with some reservations that I’m working on) that the mind is a physical operation of the brain; that we are meat computers.

            But some 80% of Americans (see the statistics Jerry always has at hand) think instead that their true essence is as a soul inhabiting an Earthly body for a bit before rejoining Jesus in Heaven. The body is but a suit they wear for a few score years before casting it off like a raggedy old suit.

            That you can hear somebody’s heart beating through a shirt is not at all surprising, and doesn’t in any way indicate that the shirt has a beating heart.

            Look at it through the perspective of a True Christian, one who cheerfully recites the Credo and actually believes it, and you’ll see how those results are perfectly compatible with soulful dualism, and, indeed, exactly the results you’d expect.

            Cheers,

            b&

          55. No, Paul.

            Whatever you had for breakfast this morning…you had no more choice of what to eat than the Sun did whether to rise or not. Your breakfast habits are a bit more difficult to predict than the Sun’s, but your choice and the Sun’s was the same: none whatsoever.

            Cheers,

            b&

          56. I have been away from all this for a while, being busy, and will not attempt to deal with any problems in logic, since I am not competent in that respect, but will say that I largely agree with what paultorek is saying, so far as I understand it. What strikes me about the ‘incompatibilism’ espoused by some, including those who assume that because physical laws are working in both we cannot in the end distinguish between an avalanche and the creation of a scientific theory, is how little (if anything) issues from it, or is even asserted to issue from it. It seems largely an irrelevance. Jansenists & Jesuits. There is a silliness that seems peculiar to intellectuals at work. The craft of computer programming is not necessarily a good model for more important human activities.
            All that does seem to issue from it are repeated jokes, which are growing rather stale, about how some commenter or other has been constrained by the laws of physics to make his or her comment, and the assertion that an acceptance of a thoroughgoing determinism has, or will have, a wonderful effect on the way we punish criminals, since in future punishment will not be so much punishment as deterrence; but no attempt, so far as I remember, has been made to describe how we might devise penalties that are proportionate to particular crimes in such a way as to deter the particular offender from repeating the crime in the future as well as others who might offend in the same way.
            I find this concern with individual punishment curious, since it seems largely oblivious to, for example, the kind of social factors that have resulted in the sort of situation that has come about in Baltimore and elsewhere. It seems to me to be a peculiarly Anglo-American attitude (Britain, despite its lack of religion, has one of the highest rates of incarceration in Europe), and derives, I suspect, not so much from religion as from the sort of economic thinking, with an emphasis on the ‘individual’ as somehow separate from society, that historically has encouraged a cruel indifference to the plight of the poor (Karl Polanyi spoke of how political and economic elites ‘steeled themselves with science’ – or what they supposed was science – so that disasters such as the Irish potato famine might not be mitigated).

          57. What strikes me about the ‘incompatibilism’ espoused by some, including those who assume that because physical laws are working in both we cannot in the end distinguish between an avalanche and the creation of a scientific theory, is how little (if anything) issues from it, or is even asserted to issue from it.

            Yes. And that’s a feature, not a bug.

            There’s nothing special about humans that can be explained through appeal to magic such as “free will.”

            If you want to explain that which separates humans from other entities, you’re not going to do so with “free will.” That’s just barking up the worng tree.

            Now, if you wish to investigate consciousness, cognition, awareness, observation, and all the rest of the properties…then you’ll get useful information.

            But by pretending that, because humans are different from other entities with respect to those properties we’re somehow magically exempt from physics…well. Yeah. Of course that’s (worse than) useless.

            b&

          58. Well, I don’t know where this is going to end up, since I can’t be bothered to scroll through everything again. Thank you, Ben, for the response to paultorek and taking the troubl to respond to me: the responses make things much clearer. But as I have said before, I think the argument is not so much over ‘incompatibilism’ versus a ‘compatibilism’ that is at bottom indistinguishable from the ‘free will’ espoused by a certain kind of Christian (excluding Calvinists & Jansensists, for example) as over what might count as adequate descriptions or explanations of some categories of human, as well as (other) animal, activity. It is all very well to say, as Jerry has said, that the arguments he creates change readers’ brain-cells – well, of course they do, whether one ends up agreeing or disagreeing or even perhaps being left cold by them, and so do drugs, alcohol, listening to music, writing a thesis, having a baby, doing a scientific experiment, reading a novel, watching a play or a film, looking at a painting, being annoyed by the cat-fight going on while you are trying to study, recognising during an argument with your wife or husband that is you who are in the wrong, etc, etc… So to say this is not, it seems to me, to say very much. So that I am glad to see you saying that it is a feature and not a bug that nothing issues from the fact of a universal determinism.

          59. It is all very well to say, as Jerry has said, that the arguments he creates change readers’ brain-cells

            I think it might be better to think of that sort of thing, at least for discussions such as this, less as arguments changing people’s minds and more in terms of amplified resonant harmonic feedback loops and the like. There’s a pattern in one brain that triggers vocalizations (or typing or whatever) that establishes a similar pattern in another brain, which starts the process afresh. Not necessarily Richard’s idea of a meme, but not unlike it. A musician can relate; open the lid of a piano, hold the sustain pedal, and you can aim another instrument at it and the piano’s strings will sound the same notes as played into it. Add some electronics and you can get similar effects that can resonate and amplify to the limit of your resources. Do it worng, such as by holding a microphone too close to a loudspeaker, and you can blow shit up.

            Yes, there’s definitely the subjective perception of one mind reaching out and bending another to its will. But there’s also the subjective perception of the Sun moving across the solid blue dome of the sky, of the Earth being flat, of heavy things falling faster, and so on. Compatibilism amounts to insisting that the sky really can be usefully though of as a metal dome across which Helios races his chariot.

            b&

  5. I really don’t get it. What is so hard to understand about the fact that even when determinism holds, our actions still have consequences. *Especially* when determinism holds! It merely means that your decision on which action to take is also determined.

    I really don’t get it, it’s as if these people want to eat the cake and have it, too, by somehow viewing the world as deterministic, but naively excluding themselves from that picture – it reminds me of bad Hollywood time travel movie writing.

    1. There are some people who think that determinism is a problem for their own morality and that it may lead them and, fearfully, others to take a fatalistic outlook on life.

      It is a foreign concept to me, but it is a like a bad movie script that happens to be quite real for some people.

      1. An argument I’ve used is nobody questions that we modify the beliefs/behaviour/etc. of a child by various means, including explaining to them why a particular belief etc. is wrong, so why is it any different with an adult? Of course, it’s harder to change something fundamental with an adult because stuff is more fixed, but it’s certainly not impossible.

    2. “What is so hard to understand…” is that I don’t understand it.

      “I have no free choice about what I do, but what I do can still affect others.” OK… but if I’m not in control of what I do, what practical difference does it make to me if I’m not in control “Because God” or “Because Physics?”

      1. Maybe it doesn’t make a practical difference to you, but it does (at least in my view) to how we treat transgressors. I think the implications for the criminal justic system are profound.

        On a personal level, realizing that you couldn’t do otherwise has, for me at least, also tempered the amount of regret I feel, or self-recrimination. What’s the point if you couldn’t have done otherwise.

        1. I am using the concept of god loosely- as the force that created the universe, not in any other sense. I’m against harsh treatment of any creature, and it doesn’t phase me if god/physics is with me or not in this…

        2. JERRY WROTE: “n a personal level, realizing that you couldn’t do otherwise has, for me at least, also tempered the amount of regret I feel, or self-recrimination. What’s the point if you couldn’t have done otherwise.”

          Jerry, this is actually one of the problems
          I’ve had with the type of reasoning given by incompatibilists, including Sam Harris in his recent podcast.

          Despite protests to the contrary, this attitude shows an encroaching fatalism, if it is to do this “tempering of conscience” at all.

          What’s the point of regret and self-recrimination? It’s a vehicle for learning and change. As is the concept “I could have done otherwise!”

          Let’s say “Ted” committed a crime in which he robbed a lady by force, leaving her a quadriplegic. Do we really want Ted alleviating his sense of regret and self recrimination by appealing to the thought “Why feel that way, when I couldn’t have done otherwise?”

          The only way that logically would separate Ted from self recrimination IS a sort of fatalism. But consternation, regret, self-recrimination, realizations that “I COULD have done otherwise” that often compel change in people for the better!

          (Because to change your behavior, you have to think you CAN change your behavior, or at least it one hell of a lot more likely you’ll change, than if you have determined you can’t change. And to say you “can” choose an alternate path than the one you’ve been on is to say you have a choice between alternatives, which is logically equivalent to saying you had a choice “could have done otherwise” in the past, which is the logic we use to learn from past mistakes).

          I was going to reproduce quotes from Sam’s recent podcast when he was talking of how his views of free will affect his thoughts about his “enemies,” and it was virtually indistinguishable from how a fatalist would
          talk.

          These are the types of mixed messages that leave people confused about what incompatibilists really mean to say, IMO.

          1. I warmly agree with this. I remember seeing a Time Magazine cover (I’m pretty sure it was Time), showing Lieutenant Calley, after the disgraceful and politically motivated pardon by Nixon, sitting surrounded by children (American not Vietnamese) and looking, well, moderately cheerful – after all, he’d got away with it! Not much regret or self-recrimination there, I thought – and you’ve got the President of the United States to back you up in thinking regrets pointless! I am sorry, but there are things that I did in my life that I continue to regret deeply (not of course that I have done anything anywhere the level of what Calley did), and I think it is right that I should regret them and not dismiss them as having been determined and therefore not bothering about. Regrets and self-recrimination can certainly become obsessive and psychologically dangerous in some cases, but although I find it in the end difficult to feel forgiving about someone like, say, Albert Speer, I have some respect for the sorrow that by all accounts he genuinely felt for what he had done. Forgive me, but I find the idea that believing in a thorough-going determinism lets you off the hook in moral terms deeply shocking. (I use the word ‘moral’ on purpose.)

  6. The most common misconception about determinism I hear is seeing it as predeterminism. When I explain what determinism really is, these people are surprised, especially when they’ve adopted the compatibilist position and I tell them that compatibilist also are determinists. 🙂

    1. Any “compatibilist” who didn’t already know that compatibilism is fully deterministic, doesn’t know the first thing about compatibilism!

      1. True, though there’s some ambiguity in the modifier “fully.” That can be interpreted several ways. It would have to be clarified.

    2. Yeah. My observations are that, generally speaking, the compatibilists that make sense and the incompatibilists that make sense are in nearly perfect agreement on what the realities are.

      In these conversations it seems to me that both Cs and ICs make bad criticisms of each other’s arguments when feathers begin to ruffle.

      1. darrelle,

        That also fits my own observations about lots of atheist “kerfuffles” in general.
        When I see public atheists tussling on line, usually it’s between two smart individuals.
        And when I look at what each is actually saying, rather than the characterization each is making of the other, they really aren’t that far apart to begin with. But once something becomes a personality scuffle molehills start looking like mountains.

        (And, again, it seems to me this usually comes about by the way one side first re-characterizes what the other is saying, to make it more extreme, and things just sort of escalate in that fashion on both sides).

      1. Basically, what is going to happen is already decided, so it doesn’t matter what you do, you will steal that car, or die at a particular time, or fall in love with a particular person. It’s already programmed into your brain and can’t be changed.

    3. With that one word — predetermination — I think you’ve captured the essence of what makes many people unhappy with Jerry’s argument.

  7. I think the most common misunderstanding isn’t directly about determinism, but rather about what a choice ultimately is.

    “All else being equal,” you have a choice. But that’s just it — all else is never equal, so the choice never gets a chance to manifest itself.

    “All else being equal” is a very useful analytical model, but, like all models, it doesn’t even remotely have any real semblance to reality. The street map you spread on your tabletop? It’s as radically different as you can get from the actual geometry of the dynamically-changing fractally-complex surface of the oblate spheroid we live on — yet it’s still incredibly useful for when you want to get somewhere you’ve never been before.

    Compatibilists, of whatever variety, including both dualists and non-dualists, have mistraken the map for the territory and are missing the fact that, though, yes, the map is both flat and foldable, the Earth itself isn’t. And, no, waving your map collection and citing all the different places you’ve successfully navigated with it doesn’t change anything.

    b&

    1. If we’re talking about free will then there’s no such thing as a ‘dualist compatibilist.’ I think you might be mixing up compatibilism with accomodationism … or maybe something else.

      A compatibilist argues that making choices is a real thing not in spite of the fact that determinism is true, but because it’s true. It makes more sense when embedded in the laws of physics than when we imagine it isn’t.

      A hard determinist would say more or less the same thing but probably put scare quotes on the word “choices.” It’s not really that big a difference. Stop pushing free will compatibilists into the oogedy-boogedy camp.

      1. Sorry; didn’t mean to include everybody in the oogedy-boogedy camp, just to indicate that I don’t think any of the “yes, we do have free will” camps, oogedy-boogedy and otherwise, properly addresses the issue of “all else being equal.”

        b&

    2. So Ben, you rather avoided the question on a recent thread — why did you find “choice” a useful concept when you talked about a bisexual person “choosing” to have relations with either a male or a female? What did you mean by the word “choice” that you used there?

      1. Squeeze me? I’m certain I did — and that there were a number of back-and-forths with various people on that subthread.

        The short version goes back to “all else being equal.” Those who’re bisexual, all else being equal, could wind up being attracted to both John and Jane and sleeping with either or both. Those who aren’t? No other amount of “all else equal” will cause them to be attracted to whichever one lacks the proper plumbing.

        But will the bisexual person actually sleep with John or Jane or both? That depends, because all else is not equal.

        b&

        1. Sorry Ben, but I’m not sure I understand this concept “all else being equal”.

          The short version goes back to “all else being equal.” Those who’re bisexual, all else being equal, could wind up being attracted to both John and Jane and sleeping with either or both.

          First, when you say “all else being equal”, what are you *not* including in the “all else”? What is it that determines which they sleep with?

          Further, as you’ve said: “But that’s just it — all else is never equal, so the choice never gets a chance to manifest itself”, so why bring in this concept?

          Given that last statement, why did you feel it meaningful and useful to adopt the concept “choice”, and what do you actually mean by it?

          1. Sorry Ben, but I’m not sure I understand this concept “all else being equal”.

            It’s a common colloquial phrase that means exactly what it says, and summarized the concept of what pretty much everybody means when they’re confronted with a choice to be made.

            For example, “All else equal, I’d prefer an apple to an orange…but, after seeing that worm crawl out of the apple, I think I’ll go with the orange instead.”

            That little vignette should perfectly encapsulate both what it means to be presented with a choice and why you don’t really have any choice in the matter.

            b&

          2. Ben, I’m totally unsure why you’re bringing this “all else being equal” into this. You seem to be presenting it as about how a *compatibilist* thinks, but it is not, it is roughly the opposite of how a compatibilist thinks.

            To a compatibilist, everything else is *not* equal, lots of things are different, and that is exactly why our “meat computer” computes different outputs at different times.

            That is exactly what compatibilists mean by “choice”. If you think that “all else being equal” is somehow about the compatibilist position then, once again, you are simply not understanding compatibilism.

          3. To a compatibilist, everything else is *not* equal, lots of things are different, and that is exactly why our “meat computer” computes different outputs at different times.

            Exactly.

            It’s not the meat computer that determines the final outcome, but the differential factors presented to the meat computer.

            Ask a computer to tell you what 2 + 2 is, and what choice does it have but to tell you that it’s 4? If you had asked it something else, of course it would have answered something else — but, again, it has no choice there, either.

            The compatibilist position is akin to claiming that a computer has “free will” because its range of potential answers is unlimited. But, in that sense, the marionette has exactly the same type of free will, if the puppeteer is skilled enough.

            b&

          4. It’s not the meat computer that determines the final outcome, but the differential factors presented to the meat computer.

            Not true. The combination of both determines the output state. For a chess computer, both the state of the board, and the program, determine the computer’s move.

            … but, again, it has no choice there, either.

            You still have not explained why — if you think there is never any “choice” — you considered a useful concept to state that a bisexual person might make choices that a gay person wouldn’t. What did you mean by that concept “choice”?

            (Your previous replies have been something about “all else being equal”, without explaining why that is relevant, at the same time as saying all else is not equal, this making that phrase entirely irrelevant.)

            The compatibilist position is akin to claiming that a computer has “free will” because its range of potential answers is unlimited.

            No, not at all. After all this time you still don’t even understand what compatibilists mean by “free will”. It is not about the factors leading up to the desire to act.

        2. I suggest you try living in South-east or East Asia, where historically sexuality has been regarded as rather more protean and fluid than it has in the Christian West, or acquaint yourself with Ancient Greek society, or acquaint yourself with what happens in an all-male environment such as a prison: there are plenty of men who will make do with a sexual partner of the same sex when there is no alternative, and not because they are ‘really’ bi-sexual or homosexual.

          1. Tim, the very fact that fewer than 100% of the people in said societies engage in homosexual relations, coupled with all the people in our own anti-gay society who never engage in heterosexual relations, should be all you need to know that there really are people who are exclusively straight and exclusively gay, as well as a significant number who’re bisexual to one degree or another.

            Is it reasonable to suggest that “society” influences many bisexuals to adopt a superficially exclusive sexual orientation? Sure. And those’re the ones who fit your description of “plenty of men who will make do with a sexual partner of the same sex when there is no alternative.” They’re the textbook definition of bisexual.

            Those who don’t “make do” in either direction? Those who, even when pressured to engage in sexual relations with the undesired gender refuse to do so?

            They’re not bisexual, and all the protean fluidity you and the others are describing simply doesn’t apply.

            I know it can be difficult for a bisexual to understand that, no, really, somebody who’s straight or gay really, really doesn’t want to have sex with somebody who’s not of the right gender — which is why I keep coming back to the example of your grandmother. Though there’re certainly exceptions even there, pretty much everybody should be able to “grok” how basically nothing could make you want to have sex with your grandmother. And for those who’re straight or gay, all people of the undesired gender fall into the “your own grandmother” category.

            b&

          2. So… are you saying that men who rape men in prison do it because they are bisexual? I say prison rape is *not* about sex, it’s about power and control. Just like it is with women.

            BTW, everyone must be bisexual or masturbation wouldn’t work.

          3. Your question about prison rape could be answered with some statistics I don’t have access to.

            What percentage of serial rapists in prison whose victims outside of prison were exclusively of the opposite gender rape prisoners of the same gender, and what percentage instead turn celibate while without access to the opposite gender?

            I rather suspect you’ll find those percentages mirror all the others.

            b&

          4. Based on my experiences working with rape crisis organizations, I sincerely doubt it!

          5. All you are saying, Ben, is that not being bi-sexual, you are not in a position to choose between sleeping with a female person or a male person (and have therefore stoutly resisted being the post-prandial enjoyment at gay dinner parties – bully for you!). But whether it’s bi-sexuality or apples and oranges you are talking about or, you seem to be suggesting, with your mantra about ‘other things being equal’, that a choice, to be a true one, has to be something along the lines of the situation of Buridan’s ass, equally thirsty and hungry and stuck equidistantly between a pile of hay and a bucket of water, the mind nevertheless leaping to some sort of pure decision that is willed wholly freely and plumping for one alternative or the other; and suggesting that choosing to sleep with a man rather than a woman because, say, you have come across a man who attracts you or choosing to an orange over an apple, which you would normally prefer, since the apple has a worm in it – that these are not true choices because there is some reason for choosing the one person or thing rather than the other. But that is what choice is, and what it has always been taken to be. It is not something that exists in some metaphysical fantasyland.

          6. Tim, you’re muddying waters here a bit…but never mind.

            If “choice” is to have any meaning in any context, then it must exclude that which isn’t even an hypothetical possibility.

            I have no choice, for example, to flap my arms and thusly fly to the Moon.

            If you would never willingly eat an apple under any circumstances, worm or no, you can not reasonably be said to be able to choose to eat an apple. If you would only choose to eat the apple if the only other alternative is death, we would again not consider that you have a choice in the matter.

            Now, let’s walk through the possibilities and see when, if ever, “choice” actually has a role to play in the decision-making process.

            If presented with an apple and an orange, you might be the sort of person who hates oranges and loves apples. Your reply would be something like, “Oh, that’s easy. Give me the apple.” But you clearly have no choice in the matter; your innate love of apples and hate of oranges inevitably compels you to pick the apple, every time, without fail.

            I’ll skip over all the other variations on that theme you can think of and go straight to the version most in favor of compatibilists: yours of Buridan’s ass, but not quite so hopeless. You love both apples and oranges and each one presented to you appears to be a fine example. Your choice may superficially appear to be arbitrary, but, instead, it will be the inevitable result of some tipping of the scales. Maybe the last piece of fruit you had was an apple, so you’ll go with an orange to mix things up. But, again, where’s the choice in that? If the last piece of fruit you had was an orange, you’d inevitably have chosen the apple. The choice wasn’t yours to make; it was made for you by the series of events that led up to that moment in time.

            And that’s why I keep trying to stress the importance of “all else being equal” in these settings. It’s in many ways shorthand for “I’ve constructed a number of plausible imaginary virtual realities, each with its own copy of myself presented with different options. In this case, I pick the apple; the other, I pick the orange.” It subjectively feels like you could choose either…but, in reality, when the time comes to make the decision, you’re going to pick the one or the other based on factors entirely outside your control.

            You only think you have a choice between the apple and the orange because you can imagine being presented with that scenario in practically unlimited forms of variation, and, in your imagination, sometimes you pick the one and other times you pick the other. And you can instantly jump between the one imagined choice and the other with perfect flexibility.

            But, when you reach out and grab the piece of fruit, the one you actually do grab will not be one of those hypothetical could be this / could be that scenarios, and you really won’t have had any choice in the matter.

            b&

          7. Ben wrote: “If presented with an apple and an orange, you might be the sort of person who hates oranges and loves apples. Your reply would be something like, “Oh, that’s easy. Give me the apple.” But you clearly have no choice in the matter; your innate love of apples and hate of oranges inevitably compels you to pick the apple, every time, without fail.”

            That’s simply not the case.

            To every normal person, in everyday parlance, “choice” means you are capable of taking either action, or could have taken either action.

            I happen to hate green apples. If you put an orange and and apple in front of me and asked me to choose which to eat, I’ll choose the orange. BUT…if you ask “could you choose otherwise?” my answer is OF COURSE!
            Wanna see?

            Present a green apple and orange to me again and I’ll pick up the green apple and eat it.
            Whether I like it or not, it shows I’m fully capable of choosing to eat the green apple instead of the orange. There simply isn’t the inevitability as you describe it.

            If your reply is “where is the choice in that?” then you are simply baffled by what normal people mean by “choice.” Because that IS a choice as normal people apply the term. If you think that is NOT a choice, you don’t understand how the word is applied
            in it’s normal understanding.

            And the “all other things being equal” is just a re-statement of the “it would only be free if you could choose otherwise with all states of affairs/causes being the same” gambit, the folly of which has been shown here over and over. When I claim I can “choose between alternate actions” I’m claiming I have an *ability* and you *test abilities* by successive tests *over time,* under slightly varying conditions, to see that the ability is robust. You don’t..you CAN’T “test” it with the state of the universe in the same state every time – not only is it an impossible test to perform, one we never demand in any other empirical question, even if you COULD, you wouldn’t LEARN anything from that in terms of applicability to the normal operation of the universe!

            And this is also why your analysis of how we choose is flawed, and missing major pieces of the puzzle. When we deliberate, even when using our imagination to model possible scenarios, this isn’t simply in the realm of fantasy and illusion – we are cogitating on TRUTHS about our abilities and the world.
            “IF I place the water in the fridge it will make it cooler (a true fact) but IF I leave it out it will reach room temperature (true fact). Now, which do I prefer to drink right now? I’d prefer cooler water (true fact), therefore placing it in the fridge will fulfill this desire (true fact).”

            This is not illusory; just like the map is a way of conveying truth about a terrain, this manner of thinking about “what we are capable of” and “what will happen IF/THEN” is a way of yielding actual knowledge, truth, to navigate the real world.

            If I say I “could have chosen to eat the orange instead” that is a claim about my abilities in a similar-enough situation, relevant to that ability. You test the claim just as you perform any other empirical test, giving me iterations of the same form of choice. If I manage to choose otherwise each time, it vindicates the ability I claimed. If I fail to choose otherwise (e.g. choose apple instead of the orange) then it undermines my claim of being able to have chosen otherwise. It’s the only rational, viable way to test ANY claim.
            And that’s why it’s the real underpinning of our deliberations, and not the merely illusory/born-of-false-belief one you (and some other incompatiibilsts) keep depicting.

          8. Present a green apple and orange to me again and I’ll pick up the green apple and eat it. Whether I like it or not, it shows I’m fully capable of choosing to eat the green apple instead of the orange.

            No; it just shows that you’re a contrarian who’s easily manipulated by a bit of reverse psychology.

            All you’re doing is adding more nodes to the binary decision-making tree.

            Your entire “freely-willed” choice could be perfectly replaced by a flowchart with enough detail.

            Do even flowcharts have “free will”?

            b&

          9. Once again: all you are saying, Ben, is that there are factors, pertaining to both situation and the person acting, both conscious & unconscious in the latter case, which in particular circumstances incline us to act in one way or another, and that we knew before. Choices are not made in a void, and what I have described is what is ordinarily regarded as a choice. Choosing not to engage in sexual relations with someone for whatever reason is a choice (I, too, when young and pretty have, turned down, usually politely, advances by gay men, some of whom I personally liked, but not in that way), choosing an orange in preference to a grub-infested apple is a choice, the unthinking choice of an apple rather than an orange because your digestive system is fed up with oranges is a choice – just as, in her terrible situation, Sophie’s was. And, yes, if it pleases you to hear it, in the end all the factors that bear on a particular choice determine that choice. Nevertheless, they do not make it not a ‘real’ choice of the kind that you seem to be thinking of, which exists and has existed nowhere and at no time, except in the minds of people who are so taken up with the idea of logic that they fail to see reality.

          10. The problem is that what you’re describing as “choice” is, in fact, a set of static “if this then that” rules. That’s great, again, for analysis purposes…but, in reality, there will be only one this, and your actual “that” will depend entirely on the “this.”

            Coel likes to say that you’re exercising “free will” whenever you don’t have a gun to your head. The problem is that that’s simply an easy-to-understand example of a case where the scales are tipped pretty radically in a particular direction such that no other individual factor is likely to tip it another way. But what difference if the scales tilt hard and fast or only lean slightly? It’s the tilting that settles the matter.

            b&

          11. And in terms of logic, since that has been mentioned, your move from a situation where no choice is possible to a situation where an actual choice is presented does not seem very convincing. And by what right do you use the term ‘decision-making process’ when, on your own account, there is no such thing?

          12. And by what right do you use the term ‘decision-making process’ when, on your own account, there is no such thing?

            Our language is hopelessly dualistic. We simply don’t have the words to accurately describe reality in non-dualistic terms, so I do the least worst I can and don’t worry about the rest.

            b&

          13. ‘your move from a situation where no choice is possible to a situation where an actual choice is presented’ AND YOUR CLAIM THAT THE LATTER IS NO DIFFERENT FROM THE FORMER do ‘not seem very convincing…’

          14. Ben,

            “No; it just shows that you’re a contrarian who’s easily manipulated by a bit of reverse psychology.”

            Now you are moving the goalposts. You equated a desire for an apple to having a love for apples that “inevitably compels you to pick the apple, every time, without fail.”
            Essentially, “a person with a love for apples couldn’t choose otherwise.”
            Which doesn’t fit most real world scenarios, and I supplied a counter example.

            I claim I can choose otherwise. You can try and test it. I’ll demonstrate that I’m correct. And that fits the “ability to choose” as most people conceive it day in and day out. You keep evaluating all counter-examples to a question-begging criteria of your own. The version I give is coherent, and is in touch with real world behaviour; yours is not.


            Your entire “freely-willed” choice could be perfectly replaced by a flowchart with enough detail.

            Do even flowcharts have “free will”?

            The same could be said of everything in the universe, which leaves that observation inert in terms of actually dealing with the differences that arise within details about our universe.

            Ben, in the same sense, cherry-pies could be replaced by a “flowchart with enough detail.”

            Do flowcharts even HAVE flaky crust and a delicious cherry-flavored filling?

            The first statement could be trivially true; the second question you should immediately recognize as nonsense.

            The flowchart is supposed to be a description of the cherry pie. You don’t make cherry pies disappear because you can create a detailed description of cherry pies.

            Further, if your flowcharts aren’t detailed enough, or of the right character to identify the specific characteristic of a cherry pie, vs a pumpkin pie, vs a hot dog, vs a stone, frog or Porsche….then they have missed anything that is of importance about what it is to be “cherry pie.”

            What you keep doing is, when important differences within the universe are identified – e.g. someone being free or not free of specific constraints – you are jettisoning away from the details to the level at which you can’t see them. “Cherry pies exist? Hold on, don’t you know they are ultimately made of atomic particles, that can be traced to the Big Bang? We don’t see any Cherry Pies when looking at atoms, and cherry pies didn’t fly out of the Big Bang…so there aren’t really any Chery Pies!”

            This is what you are doing every time you zoom out from actual examples of “freedom” as it occurs within the context relevant to human affairs, to a Physics-Wide-View in which those concerns become invisible.
            No, you can’t find “freedom” at the level of Ultimate Physical Causes. Neither can you find “cherry pies,” “puppies” or “Good Sci-Fi Novels” and everything else. It’s just an impractical, nonsensical viewpoint to take.

          15. Now you are moving the goalposts. You equated a desire for an apple to having a love for apples that “inevitably compels you to pick the apple, every time, without fail.” Essentially, “a person with a love for apples couldn’t choose otherwise.” Which doesn’t fit most real world scenarios, and I supplied a counter example.

            Erm…the entire point of my example was of how, no matter where you put the goalposts, there’s no scoring position for compatibilists.

            And I did so by sketching the outlines of a spectrum with; “Would rather die than eat an apple,” at one end; “Would let a coin toss decide” in the middle; and “Would rather die than not eat the apple” at the other end. There’s no more choice to be made in the middle than at the ends.

            You can cite as many examples you like of how, in this circumstance you’d eat the apple but you wouldn’t in this other circumstance…and all you’re doing is confirming that it is, indeed, the circumstances that determine what you’re actually going to do when confronted with the apple.

            Pick any single real-world example you like…and, in that actual example, you’re not going to have any actual choice.

            If you’re starving, you will eat the apple; you have no other choice.

            If the apple is wormy, you’re not going to eat it; you have no other choice.

            If I’m playing reverse psychology games, you’re going to act according to my manipulations; you have no other choice.

            If I put a gun to your head, you’re going to do as I say; you have no other choice.

            If you’re truly indifferent, you’ll flip a coin and do whatever the coin tells you to; you have no other choice.

            If you’re mostly but not entirely indifferent, you’ll flip a coin and go with your slight preference regardless of what the coin says, maybe after “best two out of three” if necessary; you have no other choice.

            If multiple factors are at play, it’s the final weighting of all of them — the apple is wormy but you’re famished but I’m threatening to kill you if you eat it but you don’t think I have the balls to pull the trigger but you suspect somebody else will kill me if I don’t kill you and so on.

            It’s only when you consider the entire set of hypothetical ranges of scenarios encompassing “Would you like an apple?” that the illusion of choice appears.

            But, at the end of the day, you’re going to be offered an apple and whether or not you eat it will depend on the script that the laws of physics have, if I may be permitted a moment of literary anthropomorphization, written for you.

            We spend much of our time in that imagined hypothetical realm of infinite possibility…but that world doesn’t actually exist outside our skulls. In the real world, there is only ever one actuality despite however many possibilities you might imagine there could have been. And, because there’s only that one actuality, there isn’t actually anything to choose from.

            Or: again; free will is imaginary. It is a function of the imagination and only exists there — same as with anything else from fiction, especially the heroes and gods and magicians who do the impossible. In the land of imagination, anything is possible — and free will is entirely a denizen of the land of imagination.

            (And, I should add: it’s a particularly useful analytical tool, unlike the gods…but it’s still imaginary, just as maps are not the territory they represent.)

            b&

          16. Well, Ceiling Cat or Whatever knows where on the thread this comment is going to end up, but I want to say two things:
            1) It seems rather too convenient for Ben that our language is so riddled with error that when he specifically talks of ‘making decisions’ – or was it ‘decision-making’ (I can’t be bothered to search again for the place)? – he finds himself happily in the position to say that he doesn’t actually mean ‘making decisions’ or ‘decision-making’. Well, don’t use the words if you don’t like them.
            2) Ben is making a great deal more out of what he says than what is actually said in what he says.

            And I think that’s enough of that.

  8. As said earlier, I agree free choice does not really exist, but there are so many parameters, and often so little can influence a choice, and we generally do feel so much like we have a choice, that I would not radically reject the notion of some kind of apparently ‘free’ choice altogether.

    I also think your critics there are mistaken, determinism is nothing like what they propose.

    I fail to see why ‘Explorer’ is particularly rude (admittedly I only read the parts you posted, my internet connection here is too weak to go to his actual post – I’m already grateful I can get to this site). Why is he so rude and uncivil to deserve to be cut out? Not really civil ok, but not really rude either, IMMO. Could he not be redeemable?

    1. I have no wish to opine about whether or not Explorer deserved to be shown the door. But it sounds like you might be soliciting opinions on whether or not he was rude and uncivil in his comment? Yes, he was pretty rude and uncivil. It seemed to come across loud and clear to me, and rather cliche. I think I even rolled my eyes.

      1. Explorer pretty much insulted every opinion piece Jerry has ever written as being hypocritical. Further, it’s Jerry’s site, so he gets to make Da Roolz. Read them if you haven’t already.

      2. I am not vitriolic to readers on this site (unless they’re unrepentant creationists). Do you understand that there’s a difference between any of us going after someone else, and how we talk to each other as a community of readers who are supposed to respect each other? I don’t appreciate your comparison above.

        And I wasn’t asking for opinions about whether my banning someone was justified. I made the decision, and questioning it is tantamount to telling me how to run this site, which is a Roolz violation.

    2. Do me a favor, please and don’t tell me whether someone shouldn’t be banned. This person had a history of being nasty.

      Some people I consider redeemable, and give them second chances, others I don’t. I am running the site, and I make the rules. Sorry if you disagree, but that’s the way it is.

        1. These are sincere apologies.
          I didn’t know he had antecedents of uncivil behaviour, but I did not investigate. ‘My bad’ as I think you say in the USA. I think I was totally out of line there. Uncalled for and indeed against the Roolz. Sorry.

  9. Its weirder than that though! You cite “quantum indeterminacy” as if there is a cut off point where things “become quantum”. There isn’t (other than an arbitrary one! Everything is “quantum” in that everything is made up of sub-atomic particles. The issue isn’t of indeterminacy its of what it takes to be part of a moral community

    1. But wouldn’t you agree that to make significant progress towards a sound understanding of “what it takes to be part of a moral community” necessarily entails coming to a better understanding of how reality works? As in what we use all the disciplines of science to investigate?

      I am fairly sure that, generally speaking, Cs and ICs would agree with that.

    2. But things that happen at the quantum level don’t always bubble up to the macro level. A good example (though people like Depak Chopra try to argue this) is the brain. It works on classical physics.

      Now, this doesn’t mean quantum fluctuations don’t affect the derterminist world, meaning if you roll the tape back maybe there could be a variation, but it doesn’t mean everything quantum is the same as everything classical.

  10. I will be going out to supper tonight. I do not yet know what I will have to eat but have the “free will” a priori to choose anything on the menu. Is it possible, right now, at lunch time to “determine” what I will choose? No, not exactly, though if enough variables are known, it is possible to make a very accurate table of probabilities. I know where I will be going, have eaten there before and have tried most, but not all of their dishes. All have been excellent. I like all foods. I have no food allergies. I cook at home often so I tend to choose foods at restaurants when I do eat out that are too time consuming for personal preparation. I usually choose items that are more exotic with complex flavors. Last night I had beef so I probably will not choose that two nights in a row. The restaurant is moderately priced but I like a good value when making a selection. Payday is this Thursday. It is not raining now but it might sprinkle this evening. Yesterday I read an article about the value in locally sourced produce and livestock. Sometime the restaurant highlights these. My wife usually chooses fish and I end up eating some of her leftovers. I always order after her. I wrote a response to a WIET discussion today…..there are obviously countless more variables to consider that will influence my selection. Many of these variables will not be known up until the decision time has arrived. Perhaps the guy at the table next to me orders something that is particularly fantastic looking or they have a manager’s special that is too good to pass up? Bottom line is that I believe that I carry the ‘illusion’ of free will in determining my choice up until I actually make the decision, but the actual decision is absolutely determined by physics. Maybe the long period of free will ‘illusion/delusion’ makes it so hard to get fully on board with determinism? For me, the issue is settled and I am fully in Jerry’s camp.

  11. No, I don’t believe in “moral accountability,” which to me implies that one can choose freely… But I do believe in responsibility or accountability, by which I mean that a person who did a good or bad act is the entity who performed that act, and should be treated accordingly, especially for punishment.

    So you don’t believe in ‘moral accountability’ but do believe that people are accountable for their moral actions.

    If so, it seems to me that this is making a rather fine distinction. Not everyone is going to get it.

    1. I don’t know. Jerry has said it many times, and it has seemed very clear to me every time he said it. Some people won’t believe it, some people will think it is “not even wrong,” others will merely disagree and many will think it is a distinction without a difference.

      I am pretty sure that Jerry’s issue with morality in this context is the same as with the term free will. That is, dualism. Of course your conception, and many others, does not contain or imply dualism. And I have no doubt that Jerry understands that. But I think that just as ICs argue that the term free will should be abandoned because of the old baggage of dualism, Jerry is arguing that so too should the term moral accountability.

      And just as in the case of the free will arguments between Cs and ICs, the only significant difference seems to be tactical preference. There seems to be a very close agreement regarding the realities of human cognition and even what direction changes to our criminal justice systems should go in given those realities.

      1. I agree that it may simply come down to tactics. It may also come down to background.

        I’ve seen atheists argue the following:

        1.) “As an atheist, I do not believe anything. I either know or I don’t know. ‘Belief’ is a religious term, an expression of faith. No atheist should admit to believing things.”

        2.) “As an atheist, I don’t have morals. I have ethics. “Morality” is a religious invention which involves obedience to God. No atheist should agree that they have morals.”

        3.) “As an atheist, my life has no meaning or purpose. “Meaning” and “purpose” are teleological concepts. No atheist should say that their life has meaning.”

        When I come across these arguments I always wonder whether the atheist was raised in a religion where they were force-fed anti-atheist apologetics on a regular basis. “Atheism is a faith because they believe there is no God.” “If you’re an atheist then you can’t say that anything is morally right or wrong because there’s no Authority to measure against.” “If God didn’t make people for a purpose then life has no meaning and atheists should just kill themselves.” In other words, bad arguments.

        And then these children grow up and leave the church for very good reasons having to do with corruption, power, control, vice, cruelty, judgement, hypocrisy, or what have you. They become atheists.

        But they never really think through the indoctrination. Or rather, they think about it in a different way than someone coming at it from the outside. They’ve internalized apologetic propaganda as standard knowledge. And then they’re trying to be clear, consistent, and avoid what they see as a trap which other atheists too easily fall in to, one where they buy into theistic ideas.

        Is “free will” a supernatural term? Or does the phrase simply represent the act of an agent choosing for reasons and supernaturalism imported into it — or not? What makes the most sense? What causes fewer problems and confusions?

        I don’t know. But — raised without religion — I lump “free will” into the same secular philosophical category as “belief,” “morals,” and “meaning.” I used to place “spiritual” in the not-a-religious-idea category too, but changed my mind because I realized that its supernatural origins were just too fundamental and I was constantly being misinterpreted. Saying we have “free will” sounds less to me like admitting dualism and more like renouncing fatalism or the atheist-robot-boogyman. It seems like a secular term. For both rational and tactical reasons then I’m a compatibilist. Renouncing free will in order to avoid implying libertarian supernatural belief feels like atheists granting too much to religion.

        But the views are very close.

        1. I agree entirely, and don’t think we should cede words such as “morality” and “meaning” to the religious.

          It’s as though we ceded the word “life” to the vitalist and started arguing that nothing is actually alive because vitalism is wrong.

          It’s much better to make words such as “moral” naturalistic ones, which they already pretty much are.

        2. I tend to agree regarding not ceding perfectly good words to the religious. But then, I’m stubborn.

        3. That was very well said, and ditto for Coel’s comment. I think it goes beyond tactics, though. For example, there are usages of “free will” in the courts that make it clear that the term is not stamped Property of the Religious Order. When the court wants to determine whether the defendant could have understood and followed the law, they don’t usually bring in a priest.

  12. What many seem to overlook is the fact that although determinism predicts that people could not have acted other than they did, given their genes and their environment, by subjecting them and their ideas to criticism, or attempting to shame them (as appropriate), their environment changes. The change in the environment may result in a change in behavior. Isn’t that obvious?

  13. A very interesting discussion. I particularly enjoyed drakodoc’s little parable.

    Is it worth suggesting that quantum indeterminacy is not the only factor refuting predetermination? The existence of non-linear processes does that even better. If we can’t predict the weather, why should we think that all brain processes are predictable, even in principle?

    My only other thought just now, prompted by the convolutions of some discussions going on at Jason Rosenhouse’s site at the moment, is that ‘morality’ is a term that obscures more than it clarifies.

  14. The problem I see is that PCC has written many posts on determinism and free will. It has been a progression and there have been refinements. I would venture that many of those who made these ourrageous comments have only read one or two of the posts.

    Unless PCC goes back and explains every detail of all the previous posts (not possible), some people will come to uninformed conclusions.
    As far as arrogant or intemperate comments, not many people or places on the Internet, beside this web site, are civil. Ignorant, proud arrogance is taking over.

  15. As to the “no point in trying to change anyone’s behavior,” I’ve always thought that, if you believed that, why not just lie in bed and starve to death? People seem unaware that the physical process by which society changes its behavior is that elements of it are inexorably driven towards its alteration; i.e. some folks are given to proselytizing. That’s often the method by which new data is received. It’s just how it works. It’s like, why be a musician if it’s already been decided who will be a musician? Because people who become musicians are simply drawn to music. Why get up and do anything, if it’s all been decided? Because you’ll get hungry.

    As to the useless of trying to change criminals’ behaviors, it’s as useless as preparing for a volcano or tsunami. I mean, for god’s sake, we aren’t going to change the course of the tsunami, so why do anything about it? If we’re fated to die, so be it. Most of us, though, even feeling there is nothing we can do about the tsunami, feel there are things we can do to help save ourselves. Ditto, criminals; even though there’s nothing we can do to stop the world from having criminals, there are things we can do to ameliorate the situation and lessen the extent of disruptive behavior.

    The major point being that the world is deterministic and we already have figured out what to do with criminals, etc.; so the question is not how to solve these issues in a deterministic world, but rather, knowing that it is a deterministic world, how can we improve upon things?

    1. If we’re fated to die, so be it.

      We’re not fated to die. See misunderstanding #1: determinism is not fatalism. There’s no such thing as fate in the sense of a doom that can’t be avoided, no matter what you do. Because what you do does matter; you are part of the causal chain (even if all the causes in that chain are deterministic).

      If determinists want to avoid being confused with fatalists, they should stop using the language of fatalism.

  16. I find free will to be frustrating and mostly pointless. It’s frustrating because the concept is very difficult to grasp, since explanations of free will use the physics of determinism to explain why our actions are not ours, and pointless because there is so little point beyond making a technical score. It’s the ultimate in reductionism to take the fact that we are soulless meat machines and boil it down to the factoid: there is no free will. We are soulless meat machines! That has loads of implications beyond free will and it needs to be understood by everyone, and it can be but it gets side-tracked by the preoccupation with the abstraction of free will. I think free will is a PR disaster, that it turns people off of science because, “hey, if these people believe I’m not responsible for raising my own hand then all of their ideas on science and atheism are suspect.” We should focus out onto being soulless meat machines. It’s easier to understand, and a lot more appealing 😉

  17. One confusion I’m noticed a lot in these discussions is a spurious conflation of physicalism with determinism. Physicalism says that mental events supervene on physical events and have physical causes; there’s no spooky mind-stuff. Determinism says that there’s only one possible future, one outcome to any given situation. The one does not logically entail the other. And it seems that most of the time, when readers here raise their hands and say “I’m a determinist”, what they really mean is “I’m a physicalist” (or “I’m a materialist”). No philosophical commitment to a unique future is necessary to dismiss dualism and libertarian free will.

    Regarding this:

    Determinism leads you to act badly, including cheating and criminality.

    There’s some empirical evidence that telling people they’re not in control of their own actions tends to erode their motivation for self-control and thereby leads to impulsive behavior that they might later regret. Seems to me that’s reason enough to reject formulations of determinism that emphasize the notion that “you’re not in control”. Self-control is still a thing, even in a deterministic universe (if “control” means anything at all), so the goal ought to be to encourage people to exercise their self-control rather than abandoning it.

    1. The only evidence I know of has many flaws, imo, and I think they’ve been discussed before. Not being a long term study is significant, as we don’t know whether it makes a difference long term. Personally I find it doubtful, and besides, it’s morally dubious to not tell someone the truth because of how you think they will act. Perhaps the fact that you think they will behave badly is part of the reason they do behave badly for example?

      As an historian this argument has never washed with me as we already have the historical example of predestination in religion, which is similar, and ultimately did not change behaviour.

      1. I’m certainly not advocating withholding the truth. But in my view, “you’re not in control” isn’t a truth; it’s an opinion, and a dubious one, based on a bad definition of “you”.

        If physicalism is true (and it certainly seems to be), then we are our brains; there’s nothing else we can be. And it’s our brains, our “wet onboard computers” as Jerry calls them, that control our behavior; that’s what they’re for. So to say we’re not in control, it seems to me, is to deny either the identity of brain and mind, or the evolutionary purpose of having a brain in the first place.

        1. It’ interesting to me that even given physicalism, to say “we are our brains” still remains problematic. Not too the point of adducing spooky dualism, but the “problem of identity” comes in to sort of make things messy. In what sense am I “my brain?’ Am I “my brain” right now, in it’s exact current state? If so, the problem is my brain is not physically precisely the same as it was even a minute ago, let alone an hour, yesterday, last week or last year.
          And yet we are still going to call all those other brains (or versions of my brain) “me” as well. But if “I” persist over a physical substrate that constantly changes, then it seems we need a way of conceptualizing that coherently. One way is to understand what what we are identifying as “you’ and “me” are patterns over time – patterns of thoughts, behaviors, desires, etc. Patterns generated by the brain, or patterns OF brain behavior, but nonetheless the identifying aspect of what “I” am isn’t “the wet thing in my head” it’s rather the “pattern” we have coalesced into an identity.

          But then we have to ask if what “I” am is ultimately a pattern – beliefs, desires, behaviors, etc – then why does that need to be tied to a particular physical substrate? If that pattern can be produced by another physical substrate, then “I” am not identical to “the brain in my head.”

          And of course this thinking leads to trans-humanism, and all it’s philosophical questions. If we are able to build computers/robots that detect and can produce the exact same patterns as our brain can, then if we “off load” this pattern into a machine before dying, is it “me” who is now persisting? Why not? My physical substrate was changing all the time when I was a human and we still said “this is me.”

          Once the problem of identity intrudes, people will have their views and opinions, but it is a sticky subject IMO.

          1. I don’t have time right now to reply to this at length, but the sketch of an answer would be that asking whether me now is the same “me” as me 30 years ago is like asking whether the I-5 that runs through Seattle is the same as the one that runs through LA. One is continuous with the other, but there’s a lot of miles in between, so arguing about whether they’re “the same” gets you into pointless semantic hair-splitting, in my opinion.

          2. Vaal, if you like hard SF you might want to check out a novel by Greg Egan called Schild’s Ladder, which explores some of these questions about continuity of identity in a world where minds can be digitized, copied, backed up and restored, emulated on different substrates, or even transposed to entirely different physics.

          3. This is indeed an interesting issue. I think the common conceptions of “I” need to be changed. I, or you, includes all of the patterns generated by your brain, including all of those that “you” aren’t conscious of. Based on what the cognitive sciences have discovered so far the unconscious amounts to a lot more than the conscious.

            That you are not conscious of many of the processes occurring in your brain does not, in my opinion, justify the conclusion or implication that “you” are not the author of whatever behavior is topical. Including the computational processes that the words “choose” or “decide” are typically used to denote.

            For example the Libet experiments that Jerry refers to that indicate that at least certain types of decision making occur subconsciously before a person is consciously aware of it. This seems to inspire, or suggest, to many people that some degree of fatalism is warranted. That “you” are not in control. I just don’t see it. No matter when, or if, you become conscious of some aspects of the processes that constitute human decision making, it is still you that is making the decisions.

          4. Indeed, our everyday language recognizes an unconscious component to decision-making in such phrases as “Let me sleep on it” and “I just suddenly realized what I wanted”.

            So there’s really no cause for consternation when Libet and others find that things happen in our brains before we become aware of them. Spinning these results to support the claim that “you’re not in control” is irresponsible, and Dennett is right to point that out.

          5. Agreed about the dubious inferences some draw from the Libet (and similar) experiments.

            What unsettles me more is this: It’s not just that drawing an inference from the Libet experiments to conclusions like “We are not the authors of our thoughts/choices” and “we have no free will” is a dubious inference to begin with. It’s that the type of folks leaping to these conclusions are
            often either scientists, or cheerleaders for the scientific method (like we all are here).
            They know in their scientific mindset how cautious we should be in drawing conclusions, especially about understanding mysteries or complex systems. And yet they seem to become suddenly incautious and unskeptical, making huge leaps from small experiments like Libet’s, with results that are new, halting and debated, to “This Shows We Have NO Free Will! and/or “Our Thoughts Aren’t Up To Us!”

            As I’ve mentioned before, it’s like a lawyer defending someone at the head of a ponzi scheme who walks up to his client with a soft mallet, hits the clients knee to induce an involuntary response, and then declares “You see, Judge and Jury, how this experiment shows my client’s reactions are not under his conscious control – you need to conclude therefore that the ponzi scheme could not have been due to his deliberations and planning, and that he is not a responsible agent.”

            The obvious response is “Whoa, pardner, hold on there. Before we make that leap of logic with you, there’s a LOOOOT of ground to cover from that little demonstration to the complexity of behaviors and characteristics you have yet to account for.”

      2. When you say, Heather, that predestination didn’t alter behaviour, what do you mean? I’m not trying to put you on the spot, I am interested. I think of John Bunyan and his religious struggles, or of James Hogg’s wonderful novel ‘The Confessions of a Justified Sinner’. Calvinism surely changed some people’s behaviour, though Sorley MacLean has a wonderful poem – Island Funeral, I think it’s called – in which he describes the Scottish farming folk as in a deep way not accepting the doctrine of predestination when preached at a funeral of one of their own; the doctrine was too unkind.

    2. Seriously, you’re saying we should lie to people because it might erode their motivation for self-control?

      Determinism is determinism, and people are NOT in control of their own actions, at least not the way they think they are. Our notion of free agency is one of our most cherished illusion. Even so, it’s like being religious to tell people to harbor an illusion because it erodes their motivation.

      Also, those “tests” give conflicting results, so I wouldn’t rely on them were I you. Finally, do you seriously believe that determinists like Sam Harris and I have lost our motivations since we became determinists?

      1. I’m not so sure it’s a matter of lying as a matter of being careful that the truth isn’t understood incorrectly — as it all too easily is.

        There’s often a blurred line between an explanation and an excuse. There’s also a very important distinction between an after-the-fact analysis and a before-the-fact decision — a distinction which can also get blurred. Tell someone that their genes, environment, physical condition, or upbringing predisposes them to do X, let alone mandates it, and consciously or not they’ll often DO X. They expect it and they expect others expect it from them. Self-fulfilling prophesy.

        This needn’t involve free will. Consider how often teachers and parents are cautioned about revealing too much about IQ tests or statistics. We all tend to live up or down to expectations.

        I think Gregory’s problem isn’t that the general public ‘can’t handle the truth’ about free will. It’s more that they don’t interpret the truth very well.

      2. No, I’m not saying we should lie to people or tell them to harbor illusions; see my reply to Heather above.

        Since you qualify your position by saying that people are not in control “at least not the way they think they are”, may I assume that you think there is some sense in which people may be said to be in control? If so, that’s a point of agreement between us.

        And if you think that, then would you agree that saying flatly that people are not in control is at best a misleading oversimplification? Can we agree that telling people the whole truth (including the fact that experts disagree on the details) is preferable to telling misleading half-truths? That educating people about the kind of control they do have goes hand-in-hand with debunking the kind they don’t have?

  18. It seems to me, to actually arrive at determinism, incompatibilism as a notion of reality, dualism, soul, the ‘I’ have to shift stage left for there to be any room to take it in.. they cannot exist together, I think you would go mad. Arriving is the hard part as this takes some processing and difficult repackaging of one’s sense of self.
    The Professors’ continual challenge on this is good for readers, repetitive no doubt for him, and those who don’t need it but essential I feel if we are to break down our illusions, make policies and live a life based on reality, reason and compassion.
    I am loaded with self doubt on this subject but this is how I see it.

    Just an aside Professor, I am reading a book on “The coming of age of Quantum Biology”
    Life on the Edge by Jim Al-KHalili and Johnjoe McFadden. It is layman readable and very interesting for a (un)quantified novice I wondered if I could put this up as a post subject sometime in the future.

  19. Hmm. How can I tell whether an organism’s behavior as observed is a consequence of the laws of physics and the state of the universe or is generated randomly?

    Further on this point, if an organism’s behavior as observed is a consequence of the laws of physics etc. why is only what I see possible?

    I ask because of advice from game theorists to the effect that in many situations more than one action is possible and the one to take should be selected at random. If the game theorists’ advice is good (an open question) then observable behavior won’t be fully predictable.

  20. I understand fully (I think) the difference between determinism and fatalism as a matter of psychology, as a matter of an individual’s mental approach to living in a deterministic universe. But given that determinism is an “infinite regress,” how is it distinguishable from fatalism as a matter of pure physics?

    Put another way, from the standpoint of Laplace’s demon — for whom knowledge of the entire history and future of the universe is entailed by a complete understanding of the current position of every particles within it and a complete understanding of the deterministic laws of nature acting on those particles — how does a deterministic universe look any different than a fatalistic one?

    I understand (through a glass darkly, in a very non-expert way, I try to) the problems for LaPlace’s demon posed by stochastic quantum effects, by chaotic events, and by thermodynamics/entropy — how they can play havoc with the entailment of the universe’s future and past — but how does the possibility of human agency as a source of change arise, since it’s determinism all the way down, ad infinitum?

    1. Ken,
      I’m not sure I understand your question fully, but, consider the locus classicus of fatalism: the story of Oedipus Rex. In that story, I think it’s pretty clear that the gods have fated that Oedipus shall kill his father and marry his mother, and nothing he might have done will avert that fate. He does his damnedest to avoid it, and the fate still happens. Had he embraced his fate, it still would have happened. All actions lead to the same single outcome. All causal paths converge.

      That’s fatalism, and determinism doesn’t imply it in the slightest.

      1. OK, but working backward from his dread acts of patricide and incest, were not all of Oedipus’s acts fully determined by the immediately preceding state of his causal universe, which was in turn fully determined by the state immediately preceding that, ad infinitum, dating back to his birth by Jocasta?

        I understand how fatalism feels different from determinism — just as an individual’s experience of “free will” (of making conscious contra-causal choices) feels different from determinism. But since (as I accept) determinism supervenes on physics, and since (as I further accept) no dispensation from the laws of physics is available, how do determinism and fatalism differ as a matter of the inescapable laws of the universe — rather than in their psychological impact alone?

        Put another way, if libertarian free will were true, we know the laws of the universe would have to operate differently — how, in particular, they would have to suspended themselves to allow contra-causal choice. If fatalism were true, how would the laws of the universe operate differently?

        Perhaps I’m blind to an obvious distinction here, but I assure you it is not (yet, anyway) for having plucked out my own eyes in tragic despair.

        1. Ken,

          While I think some “fatalist” type descriptions or scenarios *can* look virtually indistinguishable from simply “determined,’ I think the classic-example type of fatalism is distinguished from determinism. The difference being fatalism assumes “X will happen and there is nothing I can do to stop it from happening.”

          And I think the signature difference in fatalism is introducing the purported knowledge of an outcome before it happens, and the attitude that the outcome can not be avoided.

          So in simple determinism, take a scenario where I have a choice of driving to work tomorrow or taking the subway. I know tomorrow a major road will be closed and if I choose to drive I will be snarled in slow moving traffic. Can I avoid this outcome? Yes, I can choose to take the subway instead.
          I do so, avoid the snarled traffic. The outcome was determined, but *that particular future* of my being snarled in traffic was avoidable.

          Fatalism would tend to identify a specific future scenario as unavoidable. So if I were “fated” to end up snarled in traffic, even if I knew how to avoid it, took steps to avoid it, I couldn’t avoid it; I’d end up snarled in traffic. It’s the helpless scenario, the “no choice you make can avoid X outcome.” Whereas mere determinism entails our choices alter outcomes.

          Certainly where there is the fatalism of meddling Gods ensuring an outcome, this is distinct from simple determinism. But if you remove the Gods, I think the most salient difference is this sense of “knowing before hand” the outcome, and taking the stance that whatever actions we take will not avoid it.

          So you can get a fatalistic temperament like “That guy is fated for jail, why bother trying to talk sense into him or help him avoid that outcome?”

          1. Vaal –

            I totally get the “temperament,” the “attitude,” and “assumptions” of fatalism, which is what I meant by the psychological distinction between it and determinism. But you seem to be further suggesting that, with all else being equal, the determinist can make a choice (in your example, to drive a particular route to work) that the fatalist, faced with the exact same circumstances, could not. That sounds to me like implicitly assuming the type of contra-causal free will that determinism rejects.

            I also think that the subjective experience of making choices is so firmly entrenched in ordinary human experience, that the language — and, indeed, the very idea — of “free will” is nearly impossible for homo sapiens sapiens to escape, however much we may reject its premises. I wrote something of an extended comment about this phenomenon to a post a while back, although it was kind of late in the thread, so I’m not sure how many people saw it, or maybe they saw it and skimmed over it, or maybe they just marked it down as “tl;dr”. Anyway, if you have any interest, you can find it here.

          2. Ken Kukec,

            As I mentioned, I agree that it is actually quite difficult to parse fatalism in some senses from simple determinism. (In fact, I believe some incompatibilists actually start falling into talking like fatalists).

            That’s why I try to distinguish *certain* approaches to fatalism as being easier to distinguish from simple determinism.

            Dan Dennett has made a point that goes essentially like this: Sometimes people talk in fatalistic terms by saying something like, determinism/physicalism would mean: “You can’t change the future.”

            Which is a sort of deepity. On it’s surface it sounds significant, but on closer examination it actually seems empty. Because
            …well…of course you can’t change the future. “The Future” is ultimately “whatever happens.” And “whatever happens” will happen whether determinism is true, indeterminism is true, contra-causal Libertarian free will is true, or anything else. So this doesn’t really do any work, doesn’t really make any distinction. And in a way, this is what you are left with simply from the general observation of “determinism.” Yes, a future is determined and is going to happen. But that sounds deeper than it is; in terms of relevance to our lives, you need to get more detailed than that.

            To make it something to really talk about, you have to consider not “the future” more in terms of specifics – is some “specific” future alterable or avoidable?

            If someone says “Look, you are going to do X tomorrow and there’s nothing you can do to alter that” then the discussion begins.
            But if it’s a “whatever is going to happen is going to happen” “everything is determined whatever happens” that isn’t actually as deep or relevant to our normal reasons for acting in everyday life as some seem to think.

            Anyway, as for the difference between the mere “psychological” attitude of fatalism and “actual” fatalism, again I’d make the distinction of narrowing in on a specific form of fatalism: the idea that a specific future is known, and that even knowing it, you can do nothing to avoid it.
            Hence a sense of helplessness that you would prefer another future, but no choice you make will alter the outcome; which is quite different from how we actually treat the world…because we know our actions CAN change outcomes. This is closer to the “future manipulated and held fast by fate – or by a God” type of fatalism. Nothing you do will alter God’s “plan” for you, since He will manipulate events differently in reaction to your choices, to end at the same result. (So futures containing different choices and events leading up to a final event are possible, though ALL lead to the same RESULT).

            I think the standard creeping “fatalism” some people apply in real life, “that child born in a gang ridden area won’t escape the fate of being a gang member” is to say there are too many forces conspiring to keep the future outcome the same, even if there is some variation in how the subject gets there. And it is thought so obvious a fate, that “nothing can change it,” and this is the type of fatalism determinism want to warn against.

            There is another form of fatalism, in which
            a specific future is going to happen, but it doesn’t allow for variables in the same way:
            you could be told there is a specific future, and even if you don’t want it to happen, it happens…because having been warned of that future is PART OF the chain of events that will lead you to that future.
            It’s a sort of “one path to that fate” idea, simply cataloging the causes along the way.

            That version of fatalism strikes me as perhaps indistinguishable from mere determinism.

            At least, those are my attempts to answer your question.

          3. Ken Kukec,

            Also, about this:

            “I also think that the subjective experience of making choices is so firmly entrenched in ordinary human experience, that the language — and, indeed, the very idea — of “free will” is nearly impossible for homo sapiens sapiens to escape, however much we may reject its premises.”

            As far as I can tell, that is the “free will may be false, but we seem stuck with the illusion” stance.

            I have argued against this: I think we are “stuck” with the way we think about our choice-making, thinking we could “do otherwise” NOT because it’s an illusion we can’t shake, but because we are actually thinking TRUTHS about the world – and in fact the mode of thinking we are in when deliberating is the only possible mode creatures COULD use to apprehend truths about a determined world, and navigate within it. That’s why it seems impossible to give up; not because it’s some habit or useful illusion or something.

            We conceive of “ourselves” – an identity abstracted from our experience over time – in pretty much the only way that would yield true knowledge about ourselves and allow us to predict what “we” could do “given varying circumstances.” (E.g. when I wonder about running that marathon on the weekend, predicting whether I could do so I appeal to the “me” that has run previous marathons, so my identity stretches over time). To understand how to act to fulfill a desire, I will naturally think in If/Then fashion “If I take work off that day I’ll be able to run the marathon but IF I have to work, I can’t” etc. Only one outcome can ultimately happen, and only one outcome is GOING to happen, but our rational deliberations form
            the path, and “If/Then” thinking actually is a way of yielding truth about our abilities in the world; utterly necessary in a determined world.

            What ISN’T necessary is believing we are spirits, or contra-causal. And that’s actually a reason why it doesn’t really figure that strongly in our deliberating about anything. It might be an explanation some people appeal to (a wrong one) or it might even be a sensation we have when deliberating, but it isn’t actually the important CONTENT of our deliberating. Hence we can learn we are not spirits or contra-causal, and all the really important
            things we are thinking “I COULD do this IF…or a COULD do that IF…” remains intact. What’s left over is what is actually pertinent to parsing when we have more freedom in some cases than others.

            So, again, I have always disagreed that what we are thinking during our deliberation is pure illusion; there may be some aspects of how it feels that are illusory, but the content of what we are actually thinking, or the mode, is actually truth-yielding, not illusory. That is why we can’t escape it.

          4. Vaal,

            I appreciate your thoughtful replies, and I am enjoying our exchanges. I sense there are some broad areas of agreement between us. There may also be areas of confusion — at least, or especially, on my part — that may turn out to be areas of agreement upon further clarification. And, yes, I suppose there are indeed some areas where we simply don’t see eye-to-eye at this point.

            I’m always pleased to engage in an exchange like ours on an interesting topic. But there are times, I confess, when I post a comment here on this or a related topic not knowing if anyone will respond or even read it, because writing my thoughts down forces me to think them through more thoroughly. It also makes it easier to spot a weak concept that isn’t working, one that needs to be modified or replaced. And, while I would never post anything here as a pose, something I did not truly believe in, I will sometimes take an idea that seems to have merit, or at least plausibility, and run with it to see where it leads, how it works out, if it might be worth keeping.

            I guess part of what I’m trying to say here is that I am by no means an expert in this field, either by training or occupation. I am still very much in the thinking-it-through stage. Accordingly, I don’t have deep, hard-and-fast commitments on this subject, or others related to it, and very much remain open to additional ideas, to changing my mind, to put it succinctly. Your responses have given me some additional things to think about in this regard.

            For that matter, it is very rare that I stumble upon a topic on this website that I feel even a reasonable sense of expertise on. Otherwise, I am just an amateur. (I usually try to make this clear in my comments, but sometimes strike a stronger stance — a posture sure than my own meager storehouse of knowledge and skills warrants — as a means of working through an idea, to clarify it for myself, to see if it stands up to my own analysis or to that another commenter might offer. The broad range of topics Jerry covers here, as well as the generally high quality of the regular commentariat, count for two of reasons I keep coming back to WEIT. On most of the topics encountered, I range from a somewhat knowledgeable amateur with a fair amount of background information and/or experience, to a curious seeker with a bit of background, to a rank neophyte bringing curiosity alone. I invariably find something of interest here, though, on every visit.

            Thanks again for your time and effort, Vaal, and for giving me much to think about. It’s been a pleasure, and I hope we can continue to have such exchanges.

        2. Ken,
          If classic fatalism were true, how the laws of the universe would operate differently, is that causal paths would converge. Vaal’s replies are pretty comprehensive but I thought this was worth mentioning.

          1. Thanks, Paul. I’m not certain what you mean by “causal paths would converge” (though that phrase certainly has a nice ring to it). Can you sketch out how that works scientifically — and, even better, give an example?

          2. A real-world example might be falling into a black hole. Past a certain point, there is literally nothing you can do to escape; all trajectories allowed by the laws of physics converge at the singularity.

            Similar situations arise in chess endgames, where no matter what move you choose, your opponent has a move to counter it that forces you into checkmate.

            In both examples, free will is irrelevant, because there is no available option that avoids doom.

          3. The classical and common formulation of fatalism is much different. It’s the notion that the Fates (or whichever governing deities) have decreed that you shall, for example, die at the hand of a man not born of a woman, and there’s nothing you can do to stop that. In that example, you may well successfully avoid everybody born by Caesarian section, only to be struck down by somebody cloned by way of a Star Trek transporter malfunction.

            b&

          4. So, Greg, what I hear you saying — instead of “determinism is not fatalism” — is that determinism is sometimes fatalism: at least in black holes, sometimes on chess boards, and maybe in other yet-to-be-identified locations.

            I understand the distinction between a deterministic and a fatalistic outlook: A determinist is the guy who wakes up and while shaving looks in the mirror and says the first line of the so-called “serenity prayer” (deracinated from its religious context, let us hope) — “grant me the serenity to accept the things I cannot change” (which is everything, if by “change” we mean through an act of contra-causal free will, though not if we define it otherwise).

            A fatalist, on the other hand, is the guy who shuts off his alarm in the morning, climbs back under the covers and, before dozing off again, says to himself: “Screw it; they fire me, they fire me.”

  21. This is the first time I’ve understood the basics of determinism. I guess it’s now time to go read Sam Harris’ book now that I can pony up the entry fee of basic understanding.

  22. Any given moment that is “determined” is an “effect” or “outcome” but it is also at that very moment a “cause” or “contributing event” for the next moment. It’s a perpetual never ending flow. When focusing on a particular event, e.g., a violent behavior or a gesture to help another, we are taking a snapshot of this perpetual flow of interactive events. I’ve read this posting and most of the comments while watching “Dancing with the Stars” (sorry, folks, I enjoy it) and it impressed me that this perpetual, never ending, flow is like a dance… one step, one moment, determines the next. There’s creativity, imagination; one can take a snapshot of the dance, freeze it for an instant, and yet the analysis of that snapshot would be so limited and not really give an adequate understanding of the dance itself. Well, as the saying goes “All analogies limp!” This is just my very weak effort to help me deepen my understanding of this issue.

  23. Much of the difficulty seems to me to center on the phrase “I had no choice.” Normally this is taken to claim that someone else made the decision, that it wasn’t your doing, that it wasn’t your fault.

    When it is meant to convey instead that “I didn’t have libertarian free will to suddenly not be myself for a second and do something not based on my life history to date,” it doesn’t really communicate that very clearly.

    When the professor above said “I made the decision, and questioning it is tantamount to telling me how to run this site,” we all understand that it was his choice, albeit not a libertarian free will choice, but one determined by the state of his mind at the moment it occurred.

    That’s the sense in which the meaning of the word “choice” can be preserved, and probably really has to be. Or if not, then it has to be made very clear each time the sense in which it is being used.

  24. It is a long thread, so perhaps someone noted this misconception about misconceptions:

    some things are fundamentally unpredictable if they’re affected by quantum phenomena.

    There is no such divide between classical and quantum mechanics, what I know of – deterministic chaos is fundamentally unpredictable too:

    “Chaos theory studies the behavior of dynamical systems that are highly sensitive to initial conditions—a response popularly referred to as the butterfly effect. Small differences in initial conditions (such as those due to rounding errors in numerical computation) yield widely diverging outcomes for such dynamical systems, rendering long-term prediction impossible in general.[1] This happens even though these systems are deterministic, meaning that their future behavior is fully determined by their initial conditions, with no random elements involved.[2] In other words, the deterministic nature of these systems does not make them predictable.[3][4] This behavior is known as deterministic chaos, or simply chaos.”

    [ http://en.wikipedia.org/wiki/Chaos_theory ]

    The problem is that you need to be omnipotent to predict them*, have infinite resources including infinite time (more precisely, know the initial states in exponentially diverging systems to _all_ digits of real number precision).

    Omnipotence is perhaps the philosophic definition of predictability, but not the scientific.

    The divide here is rather that quantum mechanics implies “genuine” stochasticity, a fundamentally stochastic process generates observed outcomes (from deterministically advanced states). Even infinite resources can’t predict that.

    *Perhaps it is even worse. I have only a superficial knowledge that these things exist, besides having encountered such systems once in a while (and quickly ducked out =D).

    Of course, all that, save some quantum indeterminacy, is programmed into the Universe!

    Quantum indeterminacy is also “programmed into”, being a physical mechanism of, the universe.

    Or the deterministic QM multiverse, if you want to embrace Carroll’s preferred QM theory. There is no difference between stochastic and deterministic processes in this regard, I think.

    Re “rules of morality” I checked, and morality is “the differentiation of intentions, decisions, and actions between those that are good or right and those that are bad or wrong.” [ http://en.wikipedia.org/wiki/Morality ]

    Which means you can apply ethics of jurisdiction (such as utilitarianism) to make more or less consistent rules for morality. _There_ predictability is one of the fundamentals, unless I am mistaken. Say, over here we are supposed to ‘know the whole of laws’, but in practice it means you have to predict reasonably. E.g. you don’t have to read the laws to predict that killing is bad (for the victim and associates).

    And of course not everything is regulated thusly, making folk morality a popular pastime.

  25. Well, I believe in a free will and moral accountability, even if at a rational level I know all the arguments against it. I tried to reconcile those two facts for years, but I gave up. Simply, I have no free will in this matter; no matter how many arguments I will read, deeply inside I will still hold to my views.

    But it gives you a certain – I lack here an English word – a serenitude?, that knowledge that you cannot do anything about your own views.

    1. This is what you get when you click the link on the “blog:” “The domain FreeWill.com is for sale. To purchase, call… or Click here to make an offer.”

  26. You may have covered this ages ago in some other post but have you ever read Mark Twain’s “What is Man?” I came across a reference to it in relation to the notion of free will a few days ago and, allowing for a certain amount of archaic language and a few other foibles, it appears to follow your line of thinking reasonably well.

    The part that strikes me as less than happy about all this is that, although it may not be fatalism in a strict sense, it kinda feels that way for practical purposes. We’re all caught in an invisible web of circumstances and we do as well as we can under the circumstances. We wake up one day at a card table holding a hand of cards and we had no input into the deal, the choice of card game, or the other players. Steven Pinker’s better angels have only improved – if we presume they have improved and that the notion of improvement makes any sense now – because their circumstances are better, and they will all descend back into figurative hell if the circumstances change a certain way. Our choices seem free to us in at least some ways only because we cannot fully appreciate all the circumstances all the time. Of course, the only image I can come up with then is that everyone who thinks this eventually shrugs their shoulders and does whatever they were going to do anyway.

    I am reminded of Kurt Vonnegut’s “Slaughterhouse Five”:

    “I am a Tralfamadorian, seeing all time as you might see a stretch of the Rocky Mountains. All time is all time. It does not change. It does not lend itself to warnings or explanations. It simply is. Take it moment by moment, and you find that we are all, as I’ve said before, bugs in amber.”

  27. This post, succinctly detailing Jerry’s opinion on freewill, is one with which I readily concur and the comments generally accept causal determinism: they worry in the main about what room there is for human freedom of action within the Laws of Physics.

    Jerry comments (at No.18, 6.51pm):
    “Determinism is determinism, and people are NOT in control of their own actions, at least not the way they think they are.”

    I would rephrase that to read:
    Determinism is determinism yet people have a feeling of control of their own actions because the brain is operating in a conscious mode to make decisions, but that is not the determined freedom of Compatibilism, -leave alone the utter freedom of libertarianism.

    However, though my opinion is outright hard determinism, I don’t agree with Jerry’s view that, as a consequence, a major re-write of jurisprudence (Laws, Statutes and Penalties used to constrain social behaviour) is entailed.

    Towards the end of his Post Jerry writes:
    “I do believe in responsibility or accountability, by which I mean that a person who did a good or bad act is the entity who performed that act, and should be treated accordingly, especially for punishment. Punishment should, I believe, be exacted for social good, and can involve rehabilitation, deterrence, and keeping people who are dangerous out of society to prevent further damage.”
    Again I concur and add this is all in keeping with Da Roolz of Evolution which selects successful mutations over time, -which is effectively by trial and error.

    All that statement leaves to decide is what is likely to be good and bad and what are likely to be appropriate punishments,etc. Freewill v. Determinism is a small problem by comparison. It hardly matters and is a distraction from the much harder real problem.

    Sam Harris’ The Moral Landscape has attracted much adverse (even wrong) criticism, but at least he was addressing this real problem. And what better way than to propose a rational and scientific approach as religious/metaphysical grounds for moral precepts are falsified by that same science.

  28. I regard and enjoy this WEIT site as an (often large) online debating room with all contributors listening and replying in turn, referring to Original Post or to some previous comment/reply(s).

    Email notifications are received mainly chronologically. On the Page the reply/thread by-subject system inserts reply contributions often way out of time sequence, eventually severely reducing column width. It disconnects time-of-reply from where-they-appear on Page, making scrolling the only way of locating them on the Page: it makes the flow of discussion somewhat disjointed on Page and divides the “room” into smaller sub-groups.

    JAC. Is it possible and would you consider a change to a (separately numbered?) chronological flow on Page? Are there others who think this way?

Leave a Comment

Your email address will not be published. Required fields are marked *